Sie sind auf Seite 1von 39

Question.

Unanswered 1)

6) Which of the following is not considered a monoamine?

Which of the following is not considered a type of synapse?

A. B. C. D.

Dendrodendritic Axosomatic Axoaxonic Denoaxonic (Correct Answer)

A.Epinephrine B.Norepinephrine C.Dopamine D Adenosine (Correct Answer) . Question. Unanswered 7)

Which of the following amino acids can function as a neurotransmitter in the CNS? A. B. C. D.

Question. Unanswered 2) Multiple sclerosis is a disease that attacks the _______ of neurons in the CNS. A.Myelin sheaths (Correct Answer) B.Axon terminals C.Sodium channels D Nicotinic receptors . Question. Unanswered 3)

Leucine Glutamic acid (Correct Answer) Lysine Valine

Question. Unanswered 8) Valium has an effect on ____ to inhibit neural transmission. A.Epinephrine B.Norepinephrine C.GABA (Correct Answer) D Dopamine . Question. Unanswered 9)

Excessive polarization due to GABA is created due to the opening of ____ channels. A. B. C. D.

Ca++ Cl- (Correct Answer) K+ Na+

Which of the following types of cells line the ventricles and spinal cord? A. B.

Question. Unanswered 4) Huntington's chorea has been linked with a deficiency in the amino acid ______. A.Lysine B.GABA (Correct Answer) C.Valine D Tyrosine . Question. Unanswered 5)

Astrocytes Schwann cells Ependymal cells (Correct Answer) Oligodendrocytes

C. D.

Question. Unanswered 10) Schwann cells are located in the _____. A.PNS (Correct Answer) B.CNS Question. Unanswered 11)

Which of the following is a regulatory protein in the cytoplasm that helps the processes at the synapse? A. B. C. D.

Calmodulin (Correct Answer) Protein kinase Ligand Gap protein

Supporting cells located within the CNS are collectively called _____. A. B.

Neuroglia (Correct Answer) Astrocytes

Question. Unanswered

C. D.

Perikaryon Satellite cells A. B. C. D. Saltatory conduction (Correct Answer) Transmission Unmyelinated conduction Relative conduction

Question. Unanswered 12) Clostridium botulinum releases this enzyme that destroys peptide bonds. A.Amylase B.Endopeptidases (Correct Answer) C.Exopeptidases D Protein kinase . Question. Unanswered 13)

Question. Unanswered 18) Which of the following toxins blocks Sodium Channels? A.Srychnine B.Neostigmine C.Tetrodotoxin (Correct Answer) D Curare . Question. Unanswered 19)

The drug ____ blocks the reuptake of serotonin into presynaptic axons. A. B. C. D.

Prozac (Correct Answer) Valium Xanax Deprenyl

Which of the following types of cells is the most common in the CNS? A. B. C. D.

Astrocytes (Correct Answer) Oligocytes Neurocytes Celiac cells

Question. Unanswered 14) Oligodendrocytes are located in the _____. A.PNS B.CNS (Correct Answer) Question. Unanswered 15)

Question. Unanswered 20) The primary effect of cocaine on the nervous system is that cocaine blocks the re-uptake of ____. A.Monoamines (Correct Answer) B.Transamines C.Catecholamine D Monoamine oxidase .

Which of the following is not considered a catecholamine?

A. B. C. D.

Dopamine Norepinephrine Serotonin (Correct Answer) Epinephrine

Question. Unanswered 16) Myasthenia gravis is due to ____ receptors being blocked and destroyed by antibodies. A.Epinephrine B.Nicotinic C.Acetylcholine (Correct Answer) D Transient . Question. Unanswered 17)

Question. Unanswered 1) Which patient should be assigned to the traveling nurse, new to neurologic nursing care, who has been on the neurologic unit for 1 week?

The progression of a nerve impulse with the nodes of Ranvier is called _______.

A. A 34-year-old patient newly diagnosed with multiple sclerosis (MS) B. A 68-year-old patient with chronic amyotrophic lateral sclerosis (ALS)(Correct Answer) C. A 56-year-old patient with Guillain-Barre syndrome (GBS) in respiratory distress D. A 25-year-old patient admitted with CA level spinal cord injury (SCI) Explanation The traveling is relatively new to neurologic nursing and should be assigned patients whose conditions are stable and not complex. The newly diagnosed patient will need to be transferred to the ICU. The patient with C4 SCI is at risk for respiratory arrest. All three of these patients should be assigned to nurses experienced in neurologic nursing care. Focus: Assignment Question. Unanswered 2) A 23-year-old patient with a recent history of encephalitis is admitted to the medical unit with new onset generalized tonicclonic seizures. Which nursing activities included in the patients care will be best to delegate to an LPN/LVN whom you are supervising? (Choose all that apply). A.Document the onset time, nature of seizure activity, and postictal behaviors for all seizures. B.Administer phenytoin (Dilantin) 200 mg PO daily. (Correct Answer) C.Teach patient about the need for good oral hygiene. D Develop a discharge plan, including physician visits and . referral to the Epilepsy Foundation. Explanation Administration of medications is included in LPN education and scope of practice. Collection of data about the seizure activity may be accomplished by an LPN/LVN who observes initial seizure activity. An LPN/LVN would know to call the supervising RN immediately if a patient started to seize. Documentation of the seizure, patient teaching, and planning of care are complex activities that require RN level education and scope of practice. Focus: Delegation Question. Unanswered 3) You are creating a teaching plan for a patient with newly diagnosed migraine headaches. Which key items should be included in the teaching plan? (Choose all that apply).

. Explanation The experienced nursing assistant would know how to reposition the patient and how to reapply compression boots, and would remind the patient to perform activities he has been taught to perform. Assessing for redness and swelling (signs of deep venous thrombosis {DVT}) requires additional education and still appropriate to the professional nurse. Focus: Delegation/supervision Question. Unanswered 5) A nursing student is teaching a patient and family about epilepsy prior to the patients discharge. For which statement should you intervene? A. You should avoid consumption of all forms of alcohol. B. Wear you medical alert bracelet at all times. C. Protect your loved ones airway during a seizure.

D. Its OK to take over-the-counter medications. (Correct Answer) Explanation A patient with a seizure disorder should not take overthe-counter medications without consulting with the physician first. The other three statements are appropriate teaching points for patients with seizures disorders and their families. Focus: Delegation/supervision Question. Unanswered 6) You are mentoring a student nurse in the intensive care unit (ICU) while caring for a patient with meningococcal meningitis. Which action by the student requires that you intervene immediately? A.The student enters the room without putting on a mask and gown.(Correct Answer) B.The student instructs the family that visits are restricted to 10 minutes. C.The student gives the patient a warm blanket when he says he feels cold. D The student checks the patients pupil response to light . every 30 minutes. Explanation Meningococcal meningitis is spread through contact with respiratory secretions so use of a mask and gown is required to prevent spread of the infection to staff members or other patients. The other actions may not be appropriate but they do not require intervention as rapidly. The presence of a family member at the bedside may decrease patient confusion and agitation. Patients with hyperthermia frequently complain of feeling chilled, but warming the patient is not an appropriate intervention. Checking the pupil response to light is appropriate, but it is not needed every 30 minutes and is uncomfortable for a patient with photophobia. Focus: Prioritization Question. Unanswered 7) The patient who had a stroke needs to be fed. What instruction should you give to the nursing assistant who will feed the patient? A. Position the patient sitting up in bed before you feed her. (Correct Answer) B. Check the patients gag and swallowing reflexes. C. Feed the patient quickly because there are three more waiting. D. Suction the patients secretions between bites of food.

A. Avoid foods that contain tyramine, such as alcohol and aged cheese.(Missed) B. Avoid drugs such as Tagamet, nitroglycerin and Nifedipine.(Missed) C. Abortive therapy is aimed at eliminating the pain during the aura.(Missed) D. A potential side effect of medications is rebound headache.(Missed) E. Complementary therapies such as relaxation may be helpful.(Missed) F. Continue taking estrogen as prescribed by your physician. Explanation Medications such as estrogen supplements may actually trigger a migraine headache attack. All of the other statements are accurate. Focus: Prioritization Question. Unanswered 4) Which action (s) should you delegate to the experienced nursing assistant when caring for a patient with a thrombotic stroke with residual left-sided weakness? (Choose all that apply). A.Assist patient to reposition every 2 hours. (Missed) B.Reapply pneumatic compression boots. (Missed) C.Remind patient to perform active ROM. (Missed) D Check extremities for redness and edema.

Explanation Positioning the patient in a sitting position decreases the risk of aspiration. The nursing assistant is not trained to assess gag or swallowing reflexes. The patient should not be rushed during feeding. A patient who needs to be suctioned between bites of food is not handling secretions and is at risk for aspiration. This patient should be assessed further before feeding. Focus: Delegation/supervision Question. Unanswered 8) A patient recently started on phenytoin (Dilantin) to control simple complex seizures is seen in the outpatient clinic. Which information obtained during his chart review and assessment will be of greatest concern? A.The gums appear enlarged and inflamed. B.The white blood cell count is 2300/mm3. (Correct Answer) C.Patient occasionally forgets to take the phenytoin until after lunch. D Patient wants to renew his drivers license in the next . month. Explanation Leukopenia is a serious adverse effect of phenytoin and would require discontinuation of the medication. The other data indicate a need for further assessment and/or patient teaching, but will not require a change in medical treatment for the seizures. Focus: Prioritization Question. Unanswered 9) You are preparing to admit a patient with a seizure disorder. Which of the following actions can you delegate to LPN/LVN? A.

11) A patient with Parkinson disease has a nursing diagnosis of Impaired Physical Mobility related to neuromuscular impairment. You observe a nursing assistant performing all of these actions. For which action must you intervene? A. The NA assists the patient to ambulate to the bathroom and back to bed. B. The NA reminds the patient not to look at his feet when he is walking. C. The NA performs the patients complete bath and oral care. (Correct Answer) D. The NA sets up the patients tray and encourages patient to feed himself. Explanation The nursing assistant should assist the patient with morning care as needed, but the goal is to keep this patient as independent and mobile as possible. Assisting the patient to ambulate, reminding the patient not to look at his feet (to prevent falls), and encouraging the patient to feed himself are all appropriate to goal of maintaining independence. Focus: Delegation/supervision Question. Unanswered 12) A patient with a spinal cord injury at level C3-4 is being cared for in the ED. What is the priority assessment? A.Determine the level at which the patient has intact sensation. B.Assess the level at which the patient has retained mobility. C.Check blood pressure and pulse for signs of spinal shock. D Monitor respiratory effort and oxygen saturation . level. (Correct Answer) Explanation The first priority for the patient with an SCI is assessing respiratory patterns and ensuring an adequate airway. The patient with a high cervical injury is at risk for respiratory compromise because the spinal nerves (C3 5) innervate the phrenic nerve, which controls the diaphragm. The other assessments are also necessary, but not as high priority. Focus: Prioritization Question. Unanswered 13) The nurse is preparing to discharge a patient with chronic low back pain. Which statement by the patient indicates that additional teaching is necessary? A. I will avoid exercise because the pain gets worse. (Correct Answer) B. I will use heat or ice to help control the pain. C. I will not wear high-heeled shoes at home or work.

Complete admission assessment.

B. Set up oxygen and suction equipment. (Correct Answer) C. Place a padded tongue blade at bedside. D. Pad the side rails before patient arrives.

Explanation The LPN/LVN can set up the equipment for oxygen and suctioning. The RN should perform the complete initial assessment. Padded side rails are controversial in terms of whether they actually provide safety and ay embarrass the patient and family. Tongue blades should not be at the bedside and should never be inserted into the patients mouth after a seizure begins. Focus: Delegation/supervision. Question. Unanswered 10) Which of these patients in the neurologic ICU will be best to assign to an RN who has floated from the medical unit? A.A 26-year-old patient with a basilar skull structure who has clear drainage coming out of the nose B.A 42-year-old patient admitted several hours ago with a headache and diagnosed with a ruptured berry aneurysm. C.A 46-year-old patient who was admitted 48 hours ago with bacterial meningitis and has an antibiotic dose due (Correct Answer) D A 65-year-old patient with a astrocytoma who has just . returned to the unit after having a craniotomy Explanation This patient is the most stable of the patients listed. An RN from the medical unit would be familiar with administration of IV antibiotics. The other patients require assessments and care from RNs more experienced in caring for patients with neurologic diagnoses. Focus: Assignment. Question. Unanswered

D.I will purchase a firm mattress to replace my old one. Explanation Exercises are used to strengthen the back, relieve pressure on compressed nerves and protect the back from re-injury. Ice, heat, and firm mattresses are appropriate interventions for back pain. People with chronic back pain should avoid wearing high-heeled shoes at al times. Focus: Prioritization Question. Correct 14) A 70-year-old alcoholic patient with acute lethargy, confusion, and incontinence is admitted to the hospital ED. His wife tells you that he fell down the stairs about a month ago, but he didnt have a scratch afterward. She feels that he has become gradually less active and sleepier over the last 10 days or so.

Which of the following collaborative interventions will you implement first? A.Place on the hospital alcohol withdrawal protocol. B.Transfer to radiology for a CT scan. (Your Answer) C.Insert a retention catheter to straight drainage. D Give phenytoin (Dilantin) 100 mg PO. . Explanation The patients history and assessment data indicate that he may have a chronic subdural hematoma. The priority goal is to obtain a rapid diagnosis and send the patient to surgery to have the hematoma evacuated. The other interventions also should be implemented as soon as possible, but the initial nursing activities should be directed toward treatment of any intracranial lesion. Focus: Prioritization Question. Unanswered 15) The LPN/LVN, under your supervision, is providing nursing care for a patient with GBS. What observation would you instruct the LPN/LVN to report immediately? A. B. C. A. A 28-year-old newly admitted patient with spinal cord injury B. A 67-year-old patient with stroke 3 days ago and leftsided weakness(Correct Answer) C. An 85-year-old dementia patient to be transferred to long-term care today D. A 54-year-old patient with Parkinsons who needs assistance with bathing Explanation The new graduate RN who is oriented to the unit should be assigned stable, non-complex patients, such as the patient with stroke. The patient with Parkinsons disease needs assistance with bathing, which is best delegated to the nursing assistant. The patient being transferred to the nursing home and the newly admitted SCI should be assigned to experienced nurses. Focus: Assignment Question. Unanswered 18) You are providing care for a patient with an acute hemorrhage stroke. The patients husband has been reading a lot about strokes and asks why his wife did not receive alteplase. What is your best response? A.Your wife was not admitted within the time frame that alteplase is usually given. B.This drug is used primarily for patients who experience an acute heart attack. C.Alteplase dissolves clots and may cause more bleeding into your wifes brain. (Correct Answer) D Your wife had gallbladder surgery just 6 months ago . and this prevents the use of alteplase. Explanation Alteplase is a clot buster. With patient who has experienced hemorrhagic stroke, there is already bleeding into the brain. A drug like alteplase can worsen the bleeding. The other statements are also accurate about use of alteplase, but they are not pertinent to this patients diagnosis. Focus: Prioritization Question. Unanswered 19) The nursing assistant reports to you, the RN, that the patient with myasthenia gravis (MG) has an elevated temperature (102.20 F), heart rate of 120/minute, rise in blood pressure (158/94), and was incontinent off urine and stool. What is your best first action at this time?

Complaints of numbness and tingling Facial weakness and difficulty speaking Rapid heart rate of 102 beats per minute

D. Shallow respirations and decreased breath sounds (Correct Answer) Explanation The priority interventions for the patient with GBS are aimed at maintaining adequate respiratory function. These patients are risk for respiratory failure, which is urgent. The other findings are important and should be reported to the nurse, but they are not life-threatening. Focus: Prioritization, delegation/supervision Question. Unanswered 16) A patient who has been admitted to the medical unit with newonset angina also has a diagnosis of Alzheimers disease. Her husband tells you that he rarely gets a good nights sleep because he needs to be sure she does not wander during the night. He insists on checking each of the medications you give her to be sure they are the same as the ones she takes at home. Based on this information, which nursing diagnosis is most appropriate for this patient? A.Decreased Cardiac Output related to poor myocardial contractility B.Caregiver Role Strain related to continuous need for providing care(Correct Answer) C.Ineffective Therapeutic Regimen Management related to poor patient memory D Risk for Falls related to patient wandering behavior . during the night Explanation The husbands statement about lack of sleep and anxiety over whether the patient is receiving the correct medications are behaviors that support this diagnosis. There is no evidence that the patients cardiac output is decreased. The husbands statements about how he monitors the patient and his concern with medication administration indicate that the Risk for Ineffective Therapeutic Regimen Management and falls are not priorities at this time. Focus: Prioritization Question. Unanswered 17) Which patient should you, as charge nurse, assign to a new graduate RN who is orienting to the neurologic unit?

A. B. C. D.

Administer an acetaminophen suppository. Notify the physician immediately. (Correct Answer) Recheck vital signs in 1 hour. Reschedule patients physical therapy.

Explanation The changes that the nursing assistant is reporting are characteristics of myasthenia crisis, which often follows some type of infection. The patient is at risk for inadequate respiratory function. In addition to notifying the physician, the nurse should carefully monitor the patients respiratory status. The patient may need incubation and mechanical ventilation. The nurse would notify the physician before giving the suppository because there may be orders for cultures before giving acetaminophen. This patients vital signs need to be rechecked sooner than 1 hour. Rescheduling the physical therapy can be delegated to the unit clerk and is not urgent. Focus: Prioritization Question. Unanswered 20) While working in the ICU, you are assigned to care for a patient

with a seizure disorder. Which of these nursing actions will you implement first if the patient has a seizure? A.Place the patient on a non-rebreather mask will the oxygen at 15 L/minute. B.Administer lorazepam (Ativan) 1 mg IV. C.Turn the patient to the side and protect airway. (Correct Answer) D Assess level of consciousness during and immediately . after the seizure. Explanation The priority action during a generalized tonic-clonic seizure is to protect the airway. Administration of lorazepam should be the next action, since it will act rapidly to control the seizure. Although oxygen may be useful during the postictal phase, the hypoxemia during tonic-clonic seizures is caused by apnea. Checking the level of consciousness is not appropriate during the seizure, because generalized tonic-clonic seizures are associated with a loss of consciousness. Focus: Prioritization Question. Unanswered 21) All of these nursing activities are included in the care plan for a 78-year-old man with Parkinsons disease who has been referred to your home health agency. Which ones will you delegate to a nursing assistant (NA)? (Choose all that apply). A. Check for orthostatic changes in pulse and bloods pressure.(Missed) B. Monitor for improvement in tremor after levodopa (Ldopa) is given. C. Remind the patient to allow adequate time for meals. (Missed) D. Monitor for abnormal involuntary jerky movements of extremities. E. Assist the patient with prescribed strengthening exercises.(Missed) F. Adapt the patients preferred activities to his level of function. Explanation NA education and scope of practice includes taking pulse and blood pressure measurements. In addition, NAs can reinforce previous teaching or skills taught by the RN or other disciplines, such as speech or physical therapists. Evaluation of patient response to medication and development and individualizing the plan of care require RN-level education and scope of practice. Focus: Delegation Question. Unanswered 22) You are pulled from the ED to the neurologic floor. Which action should you delegate to the nursing assistant when providing nursing acre for a patient with SCI? A.Assess patients respiratory status every 4 hours. B.Take patients vital signs and record every 4 hours. (Correct Answer) C.Monitor nutritional status including calorie counts. D Have patient turn, cough, and deep breathe every 3 . hours. Explanation The nursing assistants training and education include taking and recording patients vital signs. The nursing assistant may assist with turning and repositioning the patient and may remind the patient to cough and deep breathe but does not teach the patient how to perform these actions. Assessing and monitoring patients require additional education and are appropriate to the scope of practice for professional nurses. Focus: Delegation/supervision Question. Unanswered 23)

What is the priority nursing diagnosis for a patient experiencing a migraine headache? A. Acute pain related to biologic and chemical factors (Correct Answer) B. Anxiety related to change in or threat to health status C. Hopelessness related to deteriorating physiological condition D. Risk for Side effects related to medical therapy Explanation The priority for interdisciplinary care for the patient experiencing a migraine headache is pain management. All of the other nursing diagnoses are accurate, but none of them is as urgent as the issue of pain, which is often incapacitating. Focus: Prioritization Question. Unanswered 24) You are helping the patient with an SCI to establish a bladderretraining program. What strategies may stimulate the patient to void? (Choose all that apply). A.Stroke the patients inner thigh. (Missed) B.Pull on the patients pubic hair. (Missed) C.Initiate intermittent straight catheterization. D Pour warm water over the perineum. (Missed) . E. Tap the bladder to stimulate detrusor muscle. (Missed) Explanation All of the strategies, except straight catheterization, may stimulate voiding in patients with SCI. Intermittent bladder catheterization can be used to empty the patients bladder, but it will not stimulate voiding. Focus: Prioritization Question. Unanswered 25) As the manager in a long-term-care (LTC) facility, you are in charge of developing a standard plan of care for residents with Alzheimers disease. Which of these nursing tasks is best to delegate to the LPN team leaders working in the facility? A. Check for improvement in resident memory after medication therapy is initiated. (Correct Answer) B. Use the Mini-Mental State Examination to assess residents every 6 months. C. Assist residents to toilet every 2 hours to decrease risk for urinary intolerance. D. Develop individualized activity plans after consulting with residents and family. Explanation LPN education and team leader responsibilities include checking for the therapeutic and adverse effects of medications. Changes in the residents memory would be communicated to the RN supervisor, who is responsible for overseeing the plan of care for each resident. Assessment for changes on the Mini-Mental State Examination and developing the plan of care are RN responsibilities. Assisting residents with personal care and hygiene would be delegated to nursing assistants working the LTC facility. Focus: Delegation Question. Unanswered 26) You are caring for a patient with a recurrent glioblastoma who is receiving dexamethasone (Decadron) 4 mg IV every 6 hours to relieve symptoms of right arm weakness and headache. Which assessment information concerns you the most? A.The patient does not recognize family members. (Correct Answer) B.The blood glucose level is 234 mg/dL. C.The patient complains of a continued headache. D The daily weight has increased 1 kg.

. Explanation The inability to recognize a family member is a new neurologic deficit for this patient, and indicates a possible increase in intracranial pressure (ICP). This change should be ommunicated to the physician immediately so that treatment can be initiated. The continued headache also indicates that the ICP may be elevated, but it is not a new problem. The glucose elevation and weight gain are common adverse effects of dexamethasone that may require treatment, but they are not emergencies. Focus: Prioritization Question. Unanswered 27) You are preparing a nursing care plan for the patient with SCI including the nursing diagnoses Impaired Physical Mobility and Self-Care Deficit. The patient tells you, I dont know why were doing all this. My lifes over. What additional nursing diagnosis takes priority based on this statement? A. B.

stimulation and irritation, and should be implemented as soon as possible. Focus: Prioritization Question. Correct 30) After receiving a change-of-shift report at 7:00 AM, which of these patients will you assess first? A.A 23-year-old with a migraine headache who is complaining of severe nausea associated with retching B.A 45-year-old who is scheduled for a craniotomy in 30 minutes and needs preoperative teaching C.A 59-year-old with Parkinsons disease who will need a swallowing assessment before breakfast D A 63-year-old with multiple sclerosis who has an oral . temperature of 101.80 F and flank pain (Your Answer) Explanation Urinary tract infections are a frequent complication in patient with multiple sclerosis because of the effect on bladder function. The elevated temperature and decreased breath sounds suggest that this patient may have pyelonephritis. The physician should be notified immediately so that antibiotic therapy can be started quickly. The other patients should be assessed soon, but do not have needs as urgent and this patient. Focus: Prioritization Question. Unanswered 31) The patient with a cervical SCI has been placed in fixed skeletal traction with a halo fixation device. When caring for this patient the nurse may delegate which action (s) to the LPN/LVN? (Choose all that apply). A. Check the patients skin for pressure form device. (Missed) B. Assess the patients neurologic status for changes. C. Observe the halo insertion sites for signs of infection. (Missed) D. Clean the halo insertion sites with hydrogen peroxide. (Missed) Explanation Checking and observing for signs of pressure or infection are within the scope of practice of the LPN/LVN. The LPN/LVN also has the appropriate skills for cleaning the halo insertion sites with hydrogen peroxide. Neurologic examination requires additional education and skill appropriate to the professional RN. Focus: Delegation/supervision Question. Unanswered 32) You are supervising a senior nursing student who is caring for a patient with a right hemisphere stroke. Which action by the student nurse requires that you intervene? A.The student instructs the patient to sit up straight, resulting in the patients puzzled expression. (Correct Answer) B.The student moves the patients tray to the right side of her over-bed tray. C.The student assists the patient with passive range-ofmotion (ROM) exercises. D The student combs the left side of the patients hair . when the patient combs only the right side. Explanation Patients with right cerebral hemisphere stroke often present with neglect syndrome. They lean to the left and when asked, respond that they believe they are sitting up straight. They often neglect the left side of their bodies and ignore food on the left side of their food trays. The nurse would need to remind the student of this phenomenon and discuss the appropriate interventions. Focus: Delegation/supervision Question. Unanswered 33)

Risk for Injury related to altered mobility Imbalanced Nutrition, Less Than Body Requirements

C. Impaired Adjustment to Spinal Cord Injury (Correct Answer) D. Poor Body Image related to immobilization Explanation The patients statement indicates impairment of adjustment to the limitations of the injury and indicates the need for additional counseling, teaching, and support. The other three nursing diagnoses may be appropriate to the patient with SCI, but they are not related to the patients statement. Focus: Prioritization Question. Unanswered 28) The patient with multiple sclerosis tells the nursing assistant that after physical therapy she is too tired to take a bath. What is your priority nursing diagnosis at this time? A.Fatigue related to disease state B.Activity Intolerance due to generalized weakness C.Impaired Physical Mobility related to neuromuscular impairment D Self-care Deficit related to fatigue and neuromuscular . weakness(Correct Answer) Explanation At this time, based on the patients statement, the priority is Self-Care Deficit related to fatigue after physical therapy. The other three nursing diagnoses are appropriate to a patient with MS, but they are not related to the patients statement. Focus: Prioritization Question. Unanswered 29) You have just admitted a patient with bacterial meningitis to the medical-surgical unit. The patient complains of a severe headache with photophobia and has a temperature of 102.60 F orally. Which collaborative intervention must be accomplished first? A. Administer codeine 15 mg orally for the patients headache. B. Infuse ceftriaxone (Rocephin) 2000 mg IV to treat the infection.(Correct Answer) C. Give acetaminophen (Tylenol) 650 mg orally to reduce the fever. D.Give furosemide (Lasix) 40 mg IV to decrease intracranial pressure Explanation Untreated bacterial meningitis has a mortality are approaching 100%, so rapid antibiotic treatment is essential. The other interventions will help reduce CNS

The patient with migraine headaches has a seizure. After the seizure, which action can you delegate to the nursing assistant?

b. c. d.

mannitol (Osmitrol) lidocaine (Xylocaine) furosemide (Lasix) After striking his head on a tree while falling from a

A. B. C. D.

3. Document the seizure. Perform neurologic checks. Take the patients vital signs. (Correct Answer) Restrain the patient for protection.

ladder, a young man age 18 is admitted to the emergency department. Hes unconscious and his pupils are nonreactive. Which intervention would be the most dangerous for the client? a. b. c. d. 4. Give him a barbiturate. Place him on mechanical ventilation. Perform a lumbar puncture. Elevate the head of his bed. When obtaining the health history from a male client with

Explanation Taking vital signs is within the education and scope of practice for a nursing assistant. The nurse should perform neurologic checks and document the seizure. Patients with seizures should not be restrained; however, the nurse may guide the patients movements as necessary. Focus: Delegation/supervision Question. Unanswered 34) A patient with a spinal cord injury (SCI) complains about a severe throbbing headache that suddenly started a short time ago. Assessment of the patient reveals increased blood pressure (168/94) and decreased heart rate (48/minute), diaphoresis, and flushing of the face and neck. What action should you take first? A.Administer the ordered acetaminophen (Tylenol). B.Check the Foley tubing for kinks or obstruction. (Correct Answer) C.Adjust the temperature in the patients room. D Notify the physician about the change in status. . Explanation These signs and symptoms are characteristic of autonomic dysreflexia, a neurologic emergency that must be promptly treated to prevent a hypertensive stroke. The cause of this syndrome is noxious stimuli, most often a distended bladder or constipation, so checking for poor catheter drainage, bladder distention, or fecal impaction is the first action that should be taken. Adjusting the room temperature may be helpful, since too cool a temperature in the room may contribute to the problem. Tylenol will not decrease the autonomic dysreflexia that is causing the patients headache. Notification of the physician may be necessary if nursing actions do not resolve symptoms. Focus: Prioritization

retinal detachment, the nurse expects the client to report: a. b. c. d. 5. light flashes and floaters in front of the eye. a recent driving accident while changing lanes. headaches, nausea, and redness of the eyes. frequent episodes of double vision. Which nursing diagnosis takes highest priority for a client

with Parkinsons crisis? a. b. c. d. 6. Imbalanced nutrition: Less than body requirements Ineffective airway clearance Impaired urinary elimination Risk for injury To encourage adequate nutritional intake for a female

client with Alzheimers disease, the nurse should: a. b. c. d. 7. stay with the client and encourage him to eat. help the client fill out his menu. give the client privacy during meals. fill out the menu for the client. The nurse is performing a mental status examination on a

male client diagnosed with subdural hematoma. This test assesses which of the following?

QUESTIONS: 1. If a male client experienced a cerebrovascular accident

a. b. c. d. 8.

Cerebellar function Intellectual function Cerebral function Sensory function Shortly after admission to an acute care facility, a male

(CVA) that damaged the hypothalamus, the nurse would anticipate that the client has problems with: a. b. c. d. 2. body temperature control. balance and equilibrium. visual acuity. thinking and reasoning. A female client admitted to an acute care facility after a

client with a seizure disorder develops status epilepticus. The physician orders diazepam (Valium) 10 mg I.V. stat. How soon can the nurse administer a second dose of diazepam, if needed and prescribed? a. b. c. d. 9. In 30 to 45 seconds In 10 to 15 minutes In 30 to 45 minutes In 1 to 2 hours A female client complains of periorbital aching, tearing,

car accident develops signs and symptoms of increased intracranial pressure (ICP). The client is intubated and placed on mechanical ventilation to help reduce ICP. To prevent a further rise in ICP caused by suctioning, the nurse anticipates administering which drug endotracheally before suctioning? a. phenytoin (Dilantin)

blurred vision, and photophobia in her right eye.

Ophthalmologic examination reveals a small, irregular, nonreactive pupil a condition resulting from acute iris inflammation (iritis). As part of the clients therapeutic regimen, the physician prescribes atropine sulfate (Atropisol), two drops of 0.5% solution in the right eye twice daily. Atropine sulfate belongs to which drug classification? a. b. c. d. 10. Parasympathomimetic agent Sympatholytic agent Adrenergic blocker Cholinergic blocker Emergency medical technicians transport a 27-year-old

13.

After an eye examination, a male client is diagnosed

with open-angle glaucoma. The physician prescribes pilocarpine ophthalmic solution (Pilocar), 0.25% gtt i, OU q.i.d. Based on this prescription, the nurse should teach the client or a family member to administer the drug by: a. instilling one drop of pilocarpine 0.25% into both eyes

daily. b. instilling one drop of pilocarpine 0.25% into both eyes

four times daily. c. instilling one drop of pilocarpine 0.25% into the right eye

daily. d. instilling one drop of pilocarpine 0.25% into the left eye

iron worker to the emergency department. They tell the nurse, He fell from a two-story building. He has a large contusion on his left chest and a hematoma in the left parietal area. He has a compound fracture of his left femur and hes comatose. We intubated him and hes maintaining an arterial oxygen saturation of 92% by pulse oximeter with a manualresuscitation bag. Which intervention by the nurse has the highest priority? a. b. c. d. 11. Assessing the left leg Assessing the pupils Placing the client in Trendelenburgs position Assessing level of consciousness An auto mechanic accidentally has battery acid splashed

four times daily. 14. A female client whos paralyzed on the left side has

been receiving physical therapy and attending teaching sessions about safety. Which behavior indicates that the client accurately understands safety measures related to paralysis? a. b. c. d. The client leaves the side rails down. The client uses a mirror to inspect the skin. The client repositions only after being reminded to do so. The client hangs the left arm over the side of the

wheelchair. 15. A male client in the emergency department has a

suspected neurologic disorder. To assess gait, the nurse asks the client to take a few steps; with each step, the clients feet make a half circle. To document the clients gait, the nurse should use which term? a. b. c. d. 16. Ataxic Dystrophic Helicopod Steppage A client, age 22, is admitted with bacterial meningitis.

in his eyes. His coworkers irrigate his eyes with water for 20 minutes, and then take him to the emergency department of a nearby hospital, where he receives emergency care for corneal injury. The physician prescribes dexamethasone (Maxidex Ophthalmic Suspension), two drops of 0.1% solution to be instilled initially into the conjunctival sacs of both eyes every hour; and polymyxin B sulfate (Neosporin Ophthalmic), 0.5% ointment to be placed in the conjunctival sacs of both eyes every 3 hours. Dexamethasone exerts its therapeutic effect by: a. b. increasing the exudative reaction of ocular tissue. decreasing leukocyte infiltration at the site of ocular

Which hospital room would be the best choice for this client? a. b. c. A private room down the hall from the nurses station An isolation room three doors from the nurses station A semiprivate room with a 32-year-old client who has viral

inflammation. c. d. inhibiting the action of carbonic anhydrase. producing a miotic reaction by stimulating and

meningitis d. A two-bed room with a client who previously had bacterial

meningitis 17. A physician diagnoses a client with myasthenia gravis,

contracting the sphincter muscles of the iris. 12. Nurse April is caring for a client who underwent a

prescribing pyridostigmine (Mestinon), 60 mg P.O. every 3 hours. Before administering this anticholinesterase agent, the nurse reviews the clients history. Which preexisting condition would contraindicate the use of pyridostigmine? a. b. c. d. Ulcerative colitis Blood dyscrasia Intestinal obstruction Spinal cord injury

lumbar laminectomy 2 days ago. Which of the following findings should the nurse consider abnormal? a. b. c. d. More back pain than the first postoperative day Paresthesia in the dermatomes near the wounds Urine retention or incontinence Temperature of 99.2 F (37.3 C)

18.

A female client is admitted to the facility for

relieve painful muscle spasms, diazepam also is recommended for: a. b. c. d. long-term treatment of epilepsy. postoperative pain management of laminectomy clients. postoperative pain management of diskectomy clients treatment of spasticity associated with spinal cord

investigation of balance and coordination problems, including possible Mnires disease. When assessing this client, the nurse expects to note: a. b. c. d. 19. vertigo, tinnitus, and hearing loss. vertigo, vomiting, and nystagmus vertigo, pain, and hearing impairment. vertigo, blurred vision, and fever. A male client with a conductive hearing disorder caused

lesions. 24. A female client who was found unconscious at home is

brought to the hospital by a rescue squad. In the intensive care unit, the nurse checks the clients oculocephalic (dolls eye) response by: a. b. c. introducing ice water into the external auditory canal. touching the cornea with a wisp of cotton. turning the clients head suddenly while holding the

by ankylosis of the stapes in the oval window undergoes a stapedectomy to remove the stapes and replace the impaired bone with a prosthesis. After the stapedectomy, the nurse should provide which client instruction? a. Lie in bed with your head elevated, and refrain from

blowing your nose for 24 hours. b. c. Try to ambulate independently after about 24 hours. Shampoo your hair every day for 10 days to help prevent

eyelids open. d. 25. shining a bright light into the pupil. While reviewing a clients chart, the nurse notices that

ear infection. d. Dont fly in an airplane, climb to high altitudes, make

the female client has myasthenia gravis. Which of the following statements about neuromuscular blocking agents is true for a client with this condition? a. The client may be less sensitive to the effects of a

sudden movements, or expose yourself to loud sounds for 30 days. 20. Nurse Oliver is monitoring a client for adverse reactions

neuromuscular blocking agent. b. Succinylcholine shouldnt be used; pancuronium may be

to dantrolene (Dantrium). Which adverse reaction is most common? a. b. c. d. 21. Excessive tearing Urine retention Muscle weakness Slurred speech The nurse is monitoring a male client for adverse

used in a lower dosage. c. Pancuronium shouldnt be used; succinylcholine may be

used in a lower dosage. d. Pancuronium and succinylcholine both require cautious

administration. 26. A male client is color blind. The nurse understands that

reactions to atropine sulfate (Atropine Care) eyedrops. Systemic absorption of atropine sulfate through the conjunctiva can cause which adverse reaction? a. b. c. d. 22. Tachycardia Increased salivation Hypotension Apnea A male client is admitted with a cervical spine injury

this client has a problem with: a. b. c. d. 27. rods. cones. lens. aqueous humor. A female client who was trapped inside a car for hours

after a head-on collision is rushed to the emergency department with multiple injuries. During the neurologic examination, the client responds to painful stimuli with decerebrate posturing. This finding indicates damage to which part of the brain? a. b. c. d. 28. Diencephalon Medulla Midbrain Cortex The nurse is assessing a 37-year-old client diagnosed

sustained during a diving accident. When planning this clients care, the nurse should assign highest priority to which nursing diagnosis? a. b. c. d. 23. Impaired physical mobility Ineffective breathing pattern Disturbed sensory perception (tactile) Self-care deficient: Dressing/grooming A male client has a history of painful, continuous muscle

spasms. He has taken several skeletal muscle relaxants without experiencing relief. His physician prescribes diazepam (Valium), 2 mg P.O. twice daily. In addition to being used to

with multiple sclerosis. Which of the following symptoms would the nurse expect to find? a. Vision changes

b. c. d. 29.

Absent deep tendon reflexes Tremors at rest Flaccid muscles The nurse is caring for a male client diagnosed with a

4.

Answer A. The sudden appearance of light flashes and

floaters in front of the affected eye is characteristic of retinal detachment. Difficulty seeing cars in another driving lane suggests gradual loss of peripheral vision, which may indicate glaucoma. Headache, nausea, and redness of the eyes are signs of acute (angle-closure) glaucoma. Double vision is common in clients with cataracts. 5. Answer B. In Parkinsons crisis, dopamine-related

cerebral aneurysm who reports a severe headache. Which action should the nurse perform? a. b. c. d. 30. Sit with the client for a few minutes. Administer an analgesic. Inform the nurse manager. Call the physician immediately. During recovery from a cerebrovascular accident (CVA),

symptoms are severely exacerbated, virtually immobilizing the client. A client confined to bed during such a crisis is at risk for aspiration and pneumonia. Also, excessive drooling increases the risk of airway obstruction. Because of these concerns, the nursing diagnosis of Ineffective airway clearance takes highest priority. Although the other options also are appropriate, they arent immediately life-threatening. 6. Answer A. Staying with the client and encouraging him to

a female client is given nothing by mouth, to help prevent aspiration. To determine when the client is ready for a liquid diet, the nurse assesses the clients swallowing ability once each shift. This assessment evaluates: a. b. c. d. cranial nerves I and II. cranial nerves III and V. cranial nerves VI and VIII. cranial nerves IX and X.

feed himself will ensure adequate food intake. A client with Alzheimers disease can forget how to eat. Allowing privacy during meals, filling out the menu, or helping the client to complete the menu doesnt ensure adequate nutritional

ANSWERS: 1. Answer A. The bodys thermostat is located in the

intake. 7. Answer C. The mental status examination assesses

functions governed by the cerebrum. Some of these are orientation, attention span, judgment, and abstract reasoning. Intellectual functioning isnt the only cerebral activity. Cerebellar function testing assesses coordination, equilibrium, and fine motor movement. Sensory function testing involves assessment of pain, light-touch sensation, and temperature discrimination. 8. Answer B. When used to treat status epilepticus,

hypothalamus; therefore, injury to that area can cause problems of body temperature control. Balance and equilibrium problems are related to cerebellar damage. Visual acuity problems would occur following occipital or optic nerve injury. Thinking and reasoning problems are the result of injury to the cerebrum. 2. Answer C. Administering lidocaine via an endotracheal

tube may minimize elevations in ICP caused by suctioning. Although mannitol and furosemide may be given to reduce ICP, theyre administered parenterally, not endotracheally. Phenytoin doesnt reduce ICP directly but may be used to abolish seizures, which can increase ICP. However, phenytoin isnt administered endotracheally. 3. Answer C. The clients history and assessment suggest

diazepam may be given every 10 to 15 minutes, as needed, to a maximum dose of 30 mg. The nurse can repeat the regimen in 2 to 4 hours, if necessary, but the total dose shouldnt exceed 100 mg in 24 hours. The nurse must not administer I.V. diazepam faster than 5 mg/minute. Therefore, the dose cant be repeated in 30 to 45 seconds because the first dose wouldnt have been administered completely by that time. Waiting longer than 15 minutes to repeat the dose would increase the clients risk of complications associated with status epilepticus. 9. Answer D. Atropine sulfate is a cholinergic blocker. It isnt

that he may have increased intracranial pressure (ICP). If this is the case, lumbar puncture shouldnt be done because it can quickly decompress the central nervous system and, thereby, cause additional damage. After a head injury, barbiturates may be given to prevent seizures; mechanical ventilation may be required if breathing deteriorates; and elevating the head of the bed may be used to reduce ICP.

a parasympathomimetic agent, a sympatholytic agent, or an adrenergic blocker. 10. Answer A. In the scenario, airway and breathing are

established so the nurses next priority should be circulation.

With a compound fracture of the femur, there is a high risk of profuse bleeding; therefore, the nurse should assess the site. Neurologic assessment is a secondary concern to airway, breathing, and circulation. The nurse doesnt have enough data to warrant putting the client in Trendelenburgs position. 11. Answer B. Dexamethasone exerts its therapeutic effect

during the initial acute phase, should be as close to the nurses station as possible to allow maximal observation. Placing the client in a room with a client who has viral meningitis may cause harm to both clients because the organisms causing viral and bacterial meningitis differ; either client may contract the others disease. Immunity to bacterial meningitis cant be acquired; therefore, a client who previously had bacterial meningitis shouldnt be put at risk by rooming with a client who has just been diagnosed with this disease. 17. Answer C. Anticholinesterase agents such as

by decreasing leukocyte infiltration at the site of ocular inflammation. This reduces the exudative reaction of diseased tissue, lessening edema, redness, and scarring. Dexamethasone and other anti-inflammatory agents dont inhibit the action of carbonic anhydrase or produce any type of miotic reaction. 12. Answer C. Urine retention or incontinence may indicate

pyridostigmine are contraindicated in a client with a mechanical obstruction of the intestines or urinary tract, peritonitis, or hypersensitivity to anticholinesterase agents. Ulcerative colitis, blood dyscrasia, and spinal cord injury dont contraindicate use of the drug. 18. Answer A. Mnires disease, an inner ear disease, is

cauda equina syndrome, which requires immediate surgery. An increase in pain on the second postoperative day is common because the long-acting local anesthetic, which may have been injected during surgery, will wear off. While paresthesia is common after surgery, progressive weakness or paralysis may indicate spinal nerve compression. A mild fever is also common after surgery but is considered significant only if it reaches 101 F (38.3 C). 13. Answer B. The abbreviation "gtt" stands for drop, "i" is

characterized by the symptom triad of vertigo, tinnitus, and hearing loss. The combination of vertigo, vomiting, and nystagmus suggests labyrinthitis. Mnires disease rarely causes pain, blurred vision, or fever. 19. Answer D. For 30 days after a stapedectomy, the client

the apothecary symbol for the number 1, OU signifies both eyes, and "q.i.d." means four times a day. Therefore, one drop of pilocarpine 0.25% should be instilled into both eyes four times daily. 14. Answer B. Using a mirror enables the client to inspect all

should avoid air travel, sudden movements that may cause trauma, and exposure to loud sounds and pressure changes (such as from high altitudes). Immediately after surgery, the client should lie flat with the surgical ear facing upward; nose blowing is permitted but should be done gently and on one side at a time. The clients first attempt at postoperative ambulation should be supervised to prevent falls caused by vertigo and light-headedness. The client must avoid shampooing and swimming to keep the dressing and the ear dry. 20. Answer C. The most common adverse reaction to

areas of the skin for signs of breakdown without the help of staff or family members. The client should keep the side rails up to help with repositioning and to prevent falls. The paralyzed client should take responsibility for repositioning or for reminding the staff to assist with it, if needed. A client with left-side paralysis may not realize that the left arm is hanging over the side of the wheelchair. However, the nurse should call this to the clients attention because the arm can get caught in the wheel spokes or develop impaired circulation from being in a dependent position for too long. 15. Answer C. A helicopod gait is an abnormal gait in which

dantrolene is muscle weakness. The drug also may depress liver function or cause idiosyncratic hepatitis. Muscle weakness is rarely severe enough to cause slurring of speech, drooling, and enuresis. Although excessive tearing and urine retention are adverse reactions associated with dantrolene use, they arent as common as muscle weakness 21. Answer A. Systemic absorption of atropine sulfate can

the clients feet make a half circle with each step. An ataxic gait is staggering and unsteady. In a dystrophic gait, the client waddles with the legs far apart. In a steppage gait, the feet and toes raise high off the floor and the heel comes down heavily with each step. 16. Answer B. A client with bacterial meningitis should be

cause tachycardia, palpitations, flushing, dry skin, ataxia, and confusion. To minimize systemic absorption, the client should apply digital pressure over the punctum at the inner canthus for 2 to 3 minutes after instilling the drops. The drug also may

kept in isolation for at least 24 hours after admission and,

cause dry mouth. It isnt known to cause hypotension or apnea. 22. Answer B. Because a cervical spine injury can cause

27.

Answer C. Decerebrate posturing, characterized by

abnormal extension in response to painful stimuli, indicates damage to the midbrain. With damage to the diencephalon or cortex, abnormal flexion (decorticate posturing) occurs when a painful stimulus is applied. Damage to the medulla results in flaccidity. 28. Answer A. Vision changes, such as diplopia, nystagmus,

respiratory distress, the nurse should take immediate action to maintain a patent airway and provide adequate oxygenation. The other options may be appropriate for a client with a spinal cord injury particularly during the course of recovery but dont take precedence over a diagnosis of Ineffective breathing pattern. 23. Answer D. In addition to relieving painful muscle

and blurred vision, are symptoms of multiple sclerosis. Deep tendon reflexes may be increased or hyperactive not absent. Babinskis sign may be positive. Tremors at rest arent characteristic of multiple sclerosis; however, intentional tremors, or those occurring with purposeful voluntary movement, are common in clients with multiple sclerosis. Affected muscles are spastic, rather than flaccid. 29. Answer D. The headache may be an indication that the

spasms, diazepam also is recommended for treatment of spasticity associated with spinal cord lesions. Diazepams use is limited by its central nervous system effects and the tolerance that develops with prolonged use. The parenteral form of diazepam can treat status epilepticus, but the drugs sedating properties make it an unsuitable choice for long-term management of epilepsy. Diazepam isnt an analgesic agent. 24. Answer C. To elicit the oculocephalic response, which

aneurysm is leaking. The nurse should notify the physician immediately. Sitting with the client is appropriate but only after the physician has been notified of the change in the clients condition. The physician will decide whether or not administration of an analgesic is indicated. Informing the nurse manager isnt necessary. 30. Answer D. Swallowing is a motor function of cranial

detects cranial nerve compression, the nurse turns the clients head suddenly while holding the eyelids open. Normally, the eyes move from side to side when the head is turned; in an abnormal response, the eyes remain fixed. The nurse introduces ice water into the external auditory canal when testing the oculovestibular response; normally, the clients eyes deviate to the side of ice water introduction. The nurse touches the clients cornea with a wisp of cotton to elicit the corneal reflex response, which reveals brain stem function; blinking is the normal response. Shining a bright light into the clients pupil helps evaluate brain stem and cranial nerve III functions; normally, the pupil responds by constricting. 25. Answer D. The nurse must cautiously administer

nerves IX and X. Cranial nerves I, II, and VIII dont possess motor functions. The motor functions of cranial nerve III include extraocular eye movement, eyelid elevation, and pupil constriction. The motor function of cranial nerve V is chewing. Cranial nerve VI controls lateral eye movement.

1.

A white female client is admitted to an acute care facility

with a diagnosis of cerebrovascular accident (CVA). Her history reveals bronchial asthma, exogenous obesity, and iron deficiency anemia. Which history finding is a risk factor for CVA? a. b. c. d. 2. Caucasian race Female sex Obesity Bronchial asthma The nurse is teaching a female client with multiple

pancuronium, succinylcholine, and any other neuromuscular blocking agent to a client with myasthenia gravis. Such a client isnt less sensitive to the effects of a neuromuscular blocking agent. Either succinylcholine or pancuronium can be administered in the usual adult dosage to a client with myasthenia gravis. 26. Answer B. Cones provide daylight color vision, and their

stimulation is interpreted as color. If one or more types of cones are absent or defective, color blindness occurs. Rods are sensitive to low levels of illumination but cant discriminate color. The lens is responsible for focusing images. Aqueous humor is a clear watery fluid and isnt involved with color perception.

sclerosis. When teaching the client how to reduce fatigue, the nurse should tell the client to: a. b. c. d. 3. take a hot bath. rest in an air-conditioned room increase the dose of muscle relaxants. avoid naps during the day A male client is having a tonic-clonic seizures. What

should the nurse do first?

a. b. c. d. 4.

Elevate the head of the bed. Restrain the clients arms and legs. Place a tongue blade in the clients mouth. Take measures to prevent injury. A female client with Guillain-Barr syndrome has

nursing diagnosis? a. b. c. d. 10. Anxiety Powerlessness Ineffective denial Risk for disuse syndrome For a male client with suspected increased intracranial

paralysis affecting the respiratory muscles and requires mechanical ventilation. When the client asks the nurse about the paralysis, how should the nurse respond? a. You may have difficulty believing this, but the paralysis

pressure (ICP), a most appropriate respiratory goal is to: a. b. c. d. prevent respiratory alkalosis. lower arterial pH. promote carbon dioxide elimination. maintain partial pressure of arterial oxygen (PaO2) above

caused by this disease is temporary. b. Youll have to accept the fact that youre permanently

paralyzed. However, you wont have any sensory loss. c. It must be hard to accept the permanency of your

80 mm Hg 11. Nurse Maureen witnesses a neighbors husband sustain

paralysis. d. 5. Youll first regain use of your legs and then your arms. The nurse is working on a surgical floor. The nurse must

a fall from the roof of his house. The nurse rushes to the victim and determines the need to opens the airway in this victim by using which method? a. b. c. d. 12. Flexed position Head tilt-chin lift Jaw thrust maneuver Modified head tilt-chin lift The nurse is assessing the motor function of an

logroll a male client following a: a. b. c. d. 6. laminectomy. thoracotomy. hemorrhoidectomy. cystectomy. A female client with a suspected brain tumor is scheduled

unconscious male client. The nurse would plan to use which plan to use which of the following to test the clients peripheral response to pain? a. b. c. d. 13. Sternal rub Nail bed pressure Pressure on the orbital rim Squeezing of the sternocleidomastoid muscle A female client admitted to the hospital with a

for computed tomography (CT). What should the nurse do when preparing the client for this test? a. Immobilize the neck before the client is moved onto a

stretcher. b. Determine whether the client is allergic to iodine,

contrast dyes, or shellfish. c. d. 7. Place a cap over the clients head. Administer a sedative as ordered. During a routine physical examination to assess a male

neurological problem asks the nurse whether magnetic resonance imaging may be done. The nurse interprets that the client may be ineligible for this diagnostic procedure based on the clients history of: a. b. c. d. 14. Hypertension Heart failure Prosthetic valve replacement Chronic obstructive pulmonary disorder A male client is having a lumbar puncture performed.

clients deep tendon reflexes, the nurse should make sure to: a. use the pointed end of the reflex hammer when striking

the Achilles tendon. b. c. d. 8. support the joint where the tendon is being tested. tap the tendon slowly and softly hold the reflex hammer tightly. A female client is admitted in a disoriented and restless

state after sustaining a concussion during a car accident. Which nursing diagnosis takes highest priority in this clients plan of care? a. b. c. d. 9. Disturbed sensory perception (visual) Self-care deficient: Dressing/grooming Impaired verbal communication Risk for injury A female client with amyotrophic lateral sclerosis (ALS)

The nurse would plan to place the client in which position? a. b. c. d. Side-lying, with a pillow under the hip Prone, with a pillow under the abdomen Prone, in slight-Trendelenburgs position Side-lying, with the legs pulled up and head bent down

onto chest. 15. The nurse is positioning the female client with

increased intracranial pressure. Which of the following positions would the nurse avoid? a. Head mildline

tells the nurse, Sometimes I feel so frustrated. I cant do anything without help! This comment best supports which

b. c. d. 16.

Head turned to the side Neck in neutral position Head of bed elevated 30 to 45 degrees A female client has clear fluid leaking from the nose

successfully if the client: a. b. c. d. 22. Gets angry with family if they interrupt a task Experiences bouts of depression and irritability Has difficulty with using modified feeding utensils Consistently uses adaptive equipment in dressing self Nurse Kristine is trying to communicate with a client

following a basilar skull fracture. The nurse assesses that this is cerebrospinal fluid if the fluid: a. b. c. d. Is clear and tests negative for glucose Is grossly bloody in appearance and has a pH of 6 Clumps together on the dressing and has a pH of 7 Separates into concentric rings and test positive of

with brain attack (stroke) and aphasia. Which of the following actions by the nurse would be least helpful to the client? a. b. c. d. 23. Speaking to the client at a slower rate Allowing plenty of time for the client to respond Completing the sentences that the client cannot finish Looking directly at the client during attempts at speech A female client has experienced an episode of

glucose 17. A male client with a spinal cord injury is prone to

experiencing automatic dysreflexia. The nurse would avoid which of the following measures to minimize the risk of recurrence? a. b. c. d. 18. Strict adherence to a bowel retraining program Keeping the linen wrinkle-free under the client Preventing unnecessary pressure on the lower limbs Limiting bladder catheterization to once every 12 hours The nurse is caring for the male client who begins to

myasthenic crisis. The nurse would assess whether the client has precipitating factors such as: a. b. c. d. 24. Getting too little exercise Taking excess medication Omitting doses of medication Increasing intake of fatty foods The nurse is teaching the female client with myasthenia

experience seizure activity while in bed. Which of the following actions by the nurse would be contraindicated? a. b. c. d. Loosening restrictive clothing Restraining the clients limbs Removing the pillow and raising padded side rails Positioning the client to side, if possible, with the head

gravis about the prevention of myasthenic and cholinergic crises. The nurse tells the client that this is most effectively done by: a. b. c. d. Eating large, well-balanced meals Doing muscle-strengthening exercises Doing all chores early in the day while less fatigued Taking medications on time to maintain therapeutic blood

flexed forward 19. The nurse is assigned to care for a female client with

levels 25. A male client with Bells palsy asks the nurse what has

complete right-sided hemiparesis. The nurse plans care knowing that this condition: a. The client has complete bilateral paralysis of the arms

caused this problem. The nurses response is based on an understanding that the cause is: a. Unknown, but possibly includes ischemia, viral infection,

and legs. b. The client has weakness on the right side of the body,

or an autoimmune problem b. Unknown, but possibly includes long-term tissue

including the face and tongue. c. The client has lost the ability to move the right arm but is

malnutrition and cellular hypoxia c. Primary genetic in origin, triggered by exposure to

able to walk independently. d. The client has lost the ability to move the right arm but is

meningitis d. Primarily genetic in origin, triggered by exposure to

able to walk independently. 20. The client with a brain attack (stroke) has residual

neurotoxins 26. The nurse has given the male client with Bells palsy

dysphagia. When a diet order is initiated, the nurse avoids doing which of the following? a. b. c. d. 21. Giving the client thin liquids Thickening liquids to the consistency of oatmeal Placing food on the unaffected side of the mouth Allowing plenty of time for chewing and swallowing The nurse is assessing the adaptation of the female

instructions on preserving muscle tone in the face and preventing denervation. The nurse determines that the client needs additional information if the client states that he or she will: a. b. c. d. Exposure to cold and drafts Massage the face with a gentle upward motion Perform facial exercises Wrinkle the forehead, blow out the cheeks, and whistle

client to changes in functional status after a brain attack (stroke). The nurse assesses that the client is adapting most

27.

Female client is admitted to the hospital with a

2.

Answer B. Fatigue is a common symptom in clients with

diagnosis of Guillain-Barre syndrome. The nurse inquires during the nursing admission interview if the client has history of: a. b. c. d. Seizures or trauma to the brain Meningitis during the last 5 years Back injury or trauma to the spinal cord Respiratory or gastrointestinal infection during the

multiple sclerosis. Lowering the body temperature by resting in an air-conditioned room may relieve fatigue; however, extreme cold should be avoided. A hot bath or shower can increase body temperature, producing fatigue. Muscle relaxants, prescribed to reduce spasticity, can cause drowsiness and fatigue. Planning for frequent rest periods and naps can relieve fatigue. Other measures to reduce fatigue in the client with multiple sclerosis include treating depression, using occupational therapy to learn energy conservation techniques, and reducing spasticity. 3. Answer D. Protecting the client from injury is the

previous month. 28. A female client with Guillian-Barre syndrome has

ascending paralysis and is intubated and receiving mechanical ventilation. Which of the following strategies would the nurse incorporate in the plan of care to help the client cope with this illness? a. Giving client full control over care decisions and

immediate priority during a seizure. Elevating the head of the bed would have no effect on the clients condition or safety. Restraining the clients arms and legs could cause injury. Placing a tongue blade or other object in the clients mouth could damage the teeth. 4. Answer A. The nurse should inform the client that the

restricting visitors b. Providing positive feedback and encouraging active range

of motion c. Providing information, giving positive feedback, and

encouraging relaxation d. Providing intravaneously administered sedatives,

paralysis that accompanies Guillain-Barr syndrome is only temporary. Return of motor function begins proximally and extends distally in the legs. 5. Answer A. The client who has had spinal surgery, such as

reducing distractions and limiting visitors 29. A male client has an impairment of cranial nerve II.

Specific to this impairment, the nurse would plan to do which of the following to ensure client to ensure client safety? a. b. c. d. 30. Speak loudly to the client Test the temperature of the shower water Check the temperature of the food on the delivery tray. Provide a clear path for ambulation without obstacles A female client has a neurological deficit involving the

laminectomy, must be logrolled to keep the spinal column straight when turning. The client who has had a thoracotomy or cystectomy may turn himself or may be assisted into a comfortable position. Under normal circumstances, hemorrhoidectomy is an outpatient procedure, and the client may resume normal activities immediately after surgery. 6. Answer B. Because CT commonly involves use of a

limbic system. Specific to this type of deficit, the nurse would document which of the following information related to the clients behavior. a. b. c. d. Is disoriented to person, place, and time Affect is flat, with periods of emotional lability Cannot recall what was eaten for breakfast today Demonstrate inability to add and subtract; does not know

contrast agent, the nurse should determine whether the client is allergic to iodine, contrast dyes, or shellfish. Neck immobilization is necessary only if the client has a suspected spinal cord injury. Placing a cap over the clients head may lead to misinterpretation of test results; instead, the hair should be combed smoothly. The physician orders a sedative only if the client cant be expected to remain still during the CT scan.

who is president

ANSWERS: 1. Answer C. Obesity is a risk factor for CVA. Other risk

7.

Answer B. To prevent the attached muscle from

contracting, the nurse should support the joint where the tendon is being tested. The nurse should use the flat, not pointed, end of the reflex hammer when striking the Achilles tendon. (The pointed end is used to strike over small areas, such as the thumb placed over the biceps tendon.) Tapping the tendon slowly and softly wouldnt provoke a deep tendon reflex response. The nurse should hold the reflex hammer

factors include a history of ischemic episodes, cardiovascular disease, diabetes mellitus, atherosclerosis of the cranial vessels, hypertension, polycythemia, smoking, hypercholesterolemia, oral contraceptive use, emotional stress, family history of CVA, and advancing age. The clients race, sex, and bronchial asthma arent risk factors for CVA.

loosely, not tightly, between the thumb and fingers so it can swing in an arc. 8. Answer D. Because the client is disoriented and restless,

14.

Answer D. The client undergoing lumbar puncture is

positioned lying on the side, with the legs pulled up to the abdomen and the head bent down onto the chest. This position helps open the spaces between the vertebrae. 15. Answer B. The head of the client with increased

the most important nursing diagnosis is risk for injury. Although the other options may be appropriate, theyre secondary because they dont immediately affect the clients health or safety. 9. Answer B. This comment best supports a nursing

intracranial pressure should be positioned so the head is in a neutral midline position. The nurse should avoid flexing or extending the clients neck or turning the head side to side. The head of the bed should be raised to 30 to 45 degrees. Use of proper positions promotes venous drainage from the cranium to keep intracranial pressure down. 16. Answer D. Leakage of cerebrospinal fluid (CSF) from the

diagnosis of Powerlessness because ALS may lead to locked-in syndrome, characterized by an active and functioning mind locked in a body that cant perform even simple daily tasks. Although Anxiety and Risk for disuse syndrome may be diagnoses associated with ALS, the clients comment specifically refers to an inability to act autonomously. A diagnosis of Ineffective denial would be indicated if the client didnt seem to perceive the personal relevance of symptoms or danger. 10. Answer C. The goal of treatment is to prevent acidemia

ears or nose may accompany basilar skull fracture. CSF can be distinguished from other body fluids because the drainage will separate into bloody and yellow concentric rings on dressing material, called a halo sign. The fluid also tests positive for glucose. 17. Answer D. The most frequent cause of autonomic

by eliminating carbon dioxide. That is because an acid environment in the brain causes cerebral vessels to dilate and therefore increases ICP. Preventing respiratory alkalosis and lowering arterial pH may bring about acidosis, an undesirable condition in this case. It isnt necessary to maintain a PaO2 as high as 80 mm Hg; 60 mm Hg will adequately oxygenate most clients. 11. Answer C. If a neck injury is suspected, the jaw thrust

dysreflexia is a distended bladder. Straight catheterization should be done every 4 to 6 hours, and foley catheters should be checked frequently to prevent kinks in the tubing. Constipation and fecal impaction are other causes, so maintaining bowel regularity is important. Other causes include stimulation of the skin from tactile, thermal, or painful stimuli. The nurse administers care to minimize risk in these areas. 18. Answer B. Nursing actions during a seizure include

maneuver is used to open the airway. The head tiltchin lift maneuver produces hyperextension of the neck and could cause complications if a neck injury is present. A flexed position is an inappropriate position for opening the airway. 12. Answer B. Motor testing in the unconscious client can be

providing for privacy, loosening restrictive clothing, removing the pillow and raising side rails in the bed, and placing the client on one side with the head flexed forward, if possible, to allow the tongue to fall forward and facilitate drainage. The limbs are never restrained because the strong muscle contractions could cause the client harm. If the client is not in bed when seizure activity begins, the nurse lowers the client to the floor, if possible, protects the head from injury, and moves furniture that may injure the client. Other aspects of care are as described for the client who is in bed. 19. Answer B. Hemiparesis is a weakness of one side of the

done only by testing response to painful stimuli. Nail bed pressure tests a basic peripheral response. Cerebral responses to pain are tested using sternal rub, placing upward pressure on the orbital rim, or squeezing the clavicle or sternocleidomastoid muscle. 13. Answer C. The client having a magnetic resonance

imaging scan has all metallic objects removed because of the magnetic field generated by the device. A careful history is obtained to determine whether any metal objects are inside the client, such as orthopedic hardware, pacemakers, artificial heart valves, aneurysm clips, or intrauterine devices. These may heat up, become dislodged, or malfunction during this procedure. The client may be ineligible if significant risk exists.

body that may occur after a stroke. Complete hemiparesis is weakness of the face and tongue, arm, and leg on one side. Complete bilateral paralysis does not occur in this condition. The client with right-sided hemiparesis has weakness of the right arm and leg and needs assistance with feeding, bathing, and ambulating.

20.

Answer A. Before the client with dysphagia is started on

is avoided. Local application of heat to the face may improve blood flow and provide comfort. 27. Answer D. Guillain-Barr syndrome is a clinical

a diet, the gag and swallow reflexes must have returned. The client is assisted with meals as needed and is given ample time to chew and swallow. Food is placed on the unaffected side of the mouth. Liquids are thickened to avoid aspiration. 21. Answer D. Clients are evaluated as coping successfully

syndrome of unknown origin that involves cranial and peripheral nerves. Many clients report a history of respiratory or gastrointestinal infection in the 1 to 4 weeks before the onset of neurological deficits. Occasionally, the syndrome can be triggered by vaccination or surgery. 28. Answer C. The client with Guillain-Barr syndrome

with lifestyle changes after a brain attack (stroke) if they make appropriate lifestyle alterations, use the assistance of others, and have appropriate social interactions. Options A, B, and C are not adaptive behaviors. 22. Answer C. Clients with aphasia after brain attack

experiences fear and anxiety from the ascending paralysis and sudden onset of the disorder. The nurse can alleviate these fears by providing accurate information about the clients condition, giving expert care and positive feedback to the client, and encouraging relaxation and distraction. The family can become involved with selected care activities and provide diversion for the client as well. 29. Answer D. Cranial nerve II is the optic nerve, which

(stroke) often fatigue easily and have a short attention span. General guidelines when trying to communicate with the aphasic client include speaking more slowly and allowing adequate response time, listening to and watching attempts to communicate, and trying to put the client at ease with a caring and understanding manner. The nurse would avoid shouting (because the client is not deaf), appearing rushed for a response, and letting family members provide all the responses for the client. 23. Answer C. Myasthenic crisis often is caused by

governs vision. The nurse can provide safety for the visually impaired client by clearing the path of obstacles when ambulating. Testing the shower water temperature would be useful if there were an impairment of peripheral nerves. Speaking loudly may help overcome a deficit of cranial nerve VIII (vestibulocochlear). Cranial nerve VII (facial) and IX (glossopharyngeal) control taste from the anterior two thirds and posterior third of the tongue, respectively. 30. Answer B. The limbic system is responsible for feelings

undermedication and responds to the administration of cholinergic medications, such as neostigmine (Prostigmin) and pyridostigmine (Mestinon). Cholinergic crisis (the opposite problem) is caused by excess medication and responds to withholding of medications. Too little exercise and fatty food intake are incorrect. Overexertion and overeating possibly could trigger myasthenic crisis. 24. Answer D. Clients with myasthenia gravis are taught to

(affect) and emotions. Calculation ability and knowledge of current events relates to function of the frontal lobe. The cerebral hemispheres, with specific regional functions, control orientation. Recall of recent events is controlled by the hippocampus.

space out activities over the day to conserve energy and restore muscle strength. Taking medications correctly to maintain blood levels that are not too low or too high is important. Muscle-strengthening exercises are not helpful and can fatigue the client. Overeating is a cause of exacerbation of symptoms, as is exposure to heat, crowds, erratic sleep habits, and emotional stress. 25. Answer A. Bells palsy is a one-sided facial paralysis

Situation: Warren was admitted to the hospital with a diagnosis of hypertension. 1. The nurse should carefully evaluate the pulse prior to administering which medication? a) clonidine (catapres) b) propanolol (inderal) c) atorvastitin calcium (lipitor) d) lovastatin (mevinolin) 2. At the time of Warren's physical examination, which finding was indicative of hypertension? a) pupil changes an opthalmoscopic exam

from compression of the facial nerve. The exact cause is unknown, but may include vascular ischemia, infection, exposure to viruses such as herpes zoster or herpes simplex, autoimmune disease, or a combination of these factors. 26. Answer A. Prevention of muscle atrophy with Bells palsy

is accomplished with facial massage, facial exercises, and electrical stimulation of the nerves. Exposure to cold or drafts

b) presence of the second heart sound c) sinus rhythm on auscultation d) cardiac electrocardiogram 3. When teaching Warren on precautions to take while on antihypertensive medication, the nurse should advice him to: a) avoid changing position suddenly b) observe for black and blue marks c) learn to take his blood pressure TID d) take the drugs always on empty stomach 4. Warren has renal damage related to his hypertensive condition. When teaching him about his diet the nurse should advice him to:

do not discontinue before surgery for pheochromocytoma

Clonidine (Catapres) is an antihypertensive drug. Although it affects both blood pressure and pulse rate so that these vital signs must be checked before administering catapres, its effect on heart rate is not as much as that of propanolol. If the patient has hypotension and bradycardia, the drug should not be given and the doctor notified.

clonidene may cause a weakly positive Coomb's test and decreases excretion of vanilymandelic acid avoid giving with propanolol and betablockers as it results in rebound hypertension avoid giving with Verapamil as it may cause AV block and severe hypotension avoid giving with herbal supplement capsicum as it may reduce antihypertensive effect of catapres avoid orthostatic hypotension by rising slowly and changing position slowly side effect drowsiness will diminish after 4 to 6 weeks the last dose should be taken immediately before going to bed advise not to discontinue drug abruptly as it may cause rebound hypertension

a) replace whole milk with milk products b) use salt substitute such as potassium chloride c) eliminate protein from his diet d) limit processed foods to fruits and juices 5. Which test should you order for Warren before treatment is indicated? a) creatinine clearance b) serum uric acid c) serum creatinine d) resting electrocardiogram

Lipitor and lovastatin are drugs used to lower LDL and total cholesterol and triglyceride levels. 2) A - letters b,c,d are examinations and findings that are more often carried out and associated with disease conditions of the heart. Hypertension is a persistent systolic blood pressure greater than 140 mmHg and a diastolic pressure greater than 90 mmHg. It is characterized by elevated peripheral vascular resistance from constriction of arterioles, which may be caused by sympathetic responses and stimulation of the renin angiotensin mechanism. It is classified as primary or essential hypertension in which there is no known etiology, and secondary, which develops as a result of some other condition. On Physical Examination:

NCLEX Secrets - Neurology Board Review: ANSWERS AND RATIONALE 1) B - Propanolol is a drug that is used for angina pectoris, MI, arrythmias, hypertension, migraine, essential tremor, pheochromocytoma. Its main effect is to block catecholamine effect in heart and blood pressure, thereby, lowering BP and heart rate. The main potential adverse effect of the drug is bradycardia, heart failure, and hypotension. Thus it is very important to always check the patient's apical pulse and blood pressure before administering the drug. If the patient has bradycardia (below 60), withhold giving the drug and notify physician.

opthalmoscopic exam: the eyes will usually reveal narrowed arterioles, hemorrhage, exudates and papilledema or swelling of the optic nerve apical and peripheral pulses vital signs and BP edema of extremities headache at the back of the head and neck nocturia confusion nausea and vomiting visual disturbances

always give with food to increase absorption advise not to discontinue abruptly as it can exacerbate angina and precipitate MI advise to continue taking the drug even he is already feeling well this drug should not be given to patients with asthma

Patients may complain of:

3) A - the most common side effect of antihypertensive drugs is orthostatic hypotension. To prevent it, instruct the patient to avoid changing position suddenly and standing for prolonged periods of time. Advise patient to sit down if he feels dizzy. 4) D - hypertensive patients without renal damage are often placed on a fat/cholesterol, low sodium and low calorie diet. Processed, preserved and fast foods are often high in sodium and must be avoided in a low sodium diet Proteins are not eliminated in the diet but its intake is limited to the recommended daily allowance to prevent overloading the kidney, adding calories and weight gain. Using salt substitutes that contain potassium may interact with the antihypertensive drugs being taken by the patient, especially when patient is taking ACE inhibitors as it may result in hyperkalemia. 5) C - the purpose of the diagnostic tests is to identify possible causes of hypertension and to identify the organs already affected by the disorder in order to institute the most effective treatment regimen for the patient. The routine laboratory tests conducted before initiating treatment include CBC, urinalysis and blood chemistry including glucose, electrolytes, cholesterol, serum creatinine and blood urea nitrogen. Serum creatinine and blood urea nitrogen reflect renal function. Hypertension can significantly decrease blood supply to the kidney which can damage the renal system and impair kidney function resulting in fluid retention and inability of the kidney to regulate electrolytes balance and excrete metabolic waste products such as urea. Hematocrit and hemoglobin are monitored as they reflect changes in fluid volume. 6. The client is diagnosed to have hypertension. He is on maintenance dose with captopril (capoten). Which of the following manifestations indicate side effects of the medication? a) diarrhea, hypoglycemia b) hyperkalemia, insomnia c) bruising, bleeding d) headache, dizziness 7. To provide exercise for a client who had cerebral vascular accident (CVA) with hemiplegia, what should the nurse include in the nursing care plan? a) provide gentle, smooth range of motion exercises, 5 times for each joint b) ensure that the arms are always above the shoulders c) provide passive flexion-extension exercises only on the stronger extremities d) massage the legs vigorously to improve muscle tone 8. When the nurse observes that the patient has extension and internal rotation of the arms and wrists and extension, plantar flexion and internal rotation of the feet, she records the patient's posturing as (supply the answer) . . . 9. Bell's palsy is a disorder of which cranial nerve? a) trigeminal (V) b) acoustic (VIII) c) facial (VII)

d) vagus (X) 10. A client diagnosed to have Guillain-Barre Syndrome. The client has the following blood gas results: pH is 7.35 and the paCO2 IS 50 mmHg. What acid-base imbalance is the client experiencing? a) respiratory acidosis b) respiratory alkalosis c) metabolic acidosis d) metabolic alkalosis

NCLEX Secrets - Neurology Board Review: ANSWERS AND RATIONALE 6) D - hypotension causes headache and dizziness. Angioedema (swelling of face and hands) may also occur as side effect of capoten. 7) A - gentle, smooth ROM exercises promote well-being of the client; prevent stiffness of joints, and contractures. Exercises should be done in a manner that prevents increase in ICP. 8) Decerebration - is abnormal extension. It indicates brainstem function impairment. 9) C - Bell's palsy is paralysis of the facial nerve (CN VII). It is characterized by asymmetry of the face with ptosis of the eyelid on the affected eye. 10) A - high paCO2 level in the blood causes respiratory acidosis. In Guillain-Barre Syndrome, there is weakness/paralysis of respiratory muscles. This causes carbon dioxide retention.

11. The nurse test for the function of abducens nerve in a client. Which of the following is appropriate nursing action? a) have the client push his arm against a resistance b) have the client shrug his shoulders c) have the client turn his eyes from right to left d) have the client smile and frown 12. Which of the following should the nurse include in the health teachings for the family of a child with meningitis? a) meningitis can be prevented by influenza vaccine b) meningitis does not pose danger to the family c) meningitis can be transferred by droplet d) meningitis is always fatal 13. A patient with cerebrovascular accident is experiencing dysphagia. Which of the following is appropriately included in the diet? a) broth b) chocolate milk c) canned tuna d) steamed pork chop

14. How does the nurse test for cranial nerve VI? a) asking the patient to close his eyes b) testing the pupillary reflexes using a penlight c) asking the patient to read with the use of Snellen chart d) with the use of penlight, move it different directions and ask the patient to follow it with his eyes 15. Which diagnostic test will be routine or regular for a patient with multiple sclerosis? a) pulmonary function test b) CT scan c) ECG d) myelography

b) with meals c) 30 minutes after meals d) at bedtime 19. A nurse should be most concerned with a patient who had closed head injury if she assessed which of the following? a) glascow coma scale rating changes from 10 to 14 b) pupil size changes from 7 mm to 5 mm c) blood pressure changes from 120/80 mmHg to 140/60 mmHg d) pulse rate changes from 90 per minute to 68 per minute 20. The client who had cerebrovascular accident is in coma. He has an advance directive which states DNR (do not resuscitate). Which of the following statements by the wife indicates effective coping? a) I should have taken him to the hospital for regular check-up b) I want to be with him when he dies c) I wish, I'm not around when he draws his last breath d) I don't want him to suffer any longer

NCLEX Secrets - Neurology Board Review: ANSWERS AND RATIONALE 11) C - the abducens innervate the lateral rectus muscles of the eyes. These muscles move the eyes from side to side (right <> left). 12) C - meningococcal infection is readily transmitted by droplet infection from nasopharyngeal secretions. 13) B - thickened liquid diet like chocolate milk or pureed foods are best tolerated by clients with dysphagia 14)D - the CN VI (abducens) innervates the lateral rectus muscles of the eyes. These muscles move the eyes from side to side. 15) A - multiple sclerosis may lead to weakness/paralysis of respiratory muscles. Assessment of pulmonary functions is necessary. 16. Which of the following nursing interventions is best for a patient withAlzheimer's disease? a) providing an unstructured environment b) providing for an environment with less decision making c) providing for flexible environment d) providing for stimulating environment 17. A patient with cerebellar impairment. Which of the following assessment is expected? a) ataxia b) apraxia c) agnosia d) agraphia 18. The client has been diagnosed to have hypertension. Captopril (capoten), an angiotensin-converting-enzyme inhibitor was prescribed by the physician. When is the best time for the client to take the medication? a) one hour before meals

NCLEX Secrets - Neurology Board Review: ANSWERS AND RATIONALE 16) B - Alzheimer's disease is characterized by loss of memory or problem with ability to think. The client with this disease will benefit most in a simple, structured environment. (Structured /routine activities/schedule). 17) A - cerebellum is responsible for balance/equilibrium. Ataxia - uncoordinated movement Apraxia - inability to perform fine motor activities Agnosia - inability to perceive sensory stimuli Agraphia - inability to write 18) A - capoten is best taken on an empty stomach to promote adequate absorption. 19) C - widening of pulse pressure (the difference between systolic pressure and diastolic pressure is more than 30 to 40 mmHg) is an indication of increased ICP. Always consider increase in ICP as emergency because it causes cerebral hypoxia. 20) B - the wife exhibits acceptance with her husband's decision. Her presence when the husband is dying will provide great psychological support to him. 21. The client had undergone hypophysectomy. Which of the following findings should the nurse give highest priority? a) rhinorrhea b) body temperature is 99F c) increased specific gravity of urine d) urine output of 40 to 50 mls/hr 22. The home health nurse is visiting a client who had spinal cord injury. Which of the following factors should the nurse

include when giving health teachings? a) regular schedule for elimination b) signs and symptoms of constipation c) proper technique of massaging the legs d) use of walker 23. What is the most appropriate nursing action to assess a patient who has spinal cord injury at the level of T6 for possible signs and symptoms of autonomic dysreflexia? a) check the body temperature b) check the blood pressure c) check the pulse rate d) check the respiratory rate 24. Which of the following problems in a client with GuillainBarre Syndrome(GBS) should be given highest priority by the nurse? a) renal problems b) neurologic problems c) respiratory problems d) cardiovascular problems 25. The client has history of seizures. He is on dilantin (phenytoin) therapy. Which of the following side effects should the nurse tell the client to report to the physician? a) reddish-brown urine b) overgrowth of gingival tissues c) drowsiness d) hyperpigmentation of the skin a) avoid over exposure to sunlight b) avoid taking pyridoxine and fortified cereals c) increase fluid intake d) discontinue the drug if it causes reddish brown discoloration of urine 27. The client is diagnosed to have amyotrophic lateral sclerosis. Which of the following signs and symptoms should the nurse observe? a) resting tremors, rigidity, shuffling gait b) muscle atrophy, spasticity, difficulty breathing c) intentional tremors, diplopia, scanning speech d) mask-like face, ascending paralysis, paresthesia 28. The client had been diagnosed to have chronic hypertension. His LDL levels are high while HDL levels are low. Which information does the nurse give to the client about factors that may increase HDL levels? a) HDL levels may be increased by physical activity and estrogen b) HDL levels may be increased by antihyperlipidemic agents c) HDL levels may be increased by low cholesterol diet d) HDL levels may be increased by bed rest 29. The client has been diagnosed to have Guillain-Barre Syndrome (GBS). Which of the following should the nurse include in the nursing care plan of the client? a) check ability to hear b) check bladder distention c) check blood pressure every 2 hours d) check deep tendon reflexes every shift 30. Who among these hypertensive clients should the nurse follow-up first? NCLEX Secrets - Neurology Board Review: ANSWERS AND RATIONALE 21) A - rhinorrhea indicates CSF leakage. Options B, C, and D are normal findings. 22) A - regular schedule of elimination is very important in a client with spinal cord injury. Bladder distention and fecal impaction may cause autonomic dysreflexia. 23) B - autonomic dysreflexia is characterized by hypertension. This is the most dangerous effect of the condition. If BP remains uncontrolled CVA may occur. 24) C - GBS involves paralysis of respiratory muscles that may lead to respiratory arrest. 25) B - gingival hyperplasia is a common toxic effect of phenytoin. Preventive measures are as follows: good oral care, use softbristled toothbrush, and massage the gums. 26. The client had been diagnosed to have Parkinson's disease. He is receiving levodopa. Which of the following health teachings should be included by the nurse? a) the white American client with BP of 160/100 mmHg, which went down to 140/90 mmHg one hour after taking Nifedipine. b) the black American with BP of 161/98 who took antihypertensive medication twice but no progress noted c) the native American client with BP ranging between 140/90 to 160/100 for the last 3 days d) the Asian American client with BP of 116/84 and claims that he had been taking anti-hypertensive medication since 2 years ago

NCLEX Secrets - Neurology Board Review: ANSWERS AND RATIONALE 26) B - pyridoxine blocks effect of levodopa. Reddish brown discoloration of urine is a harmless side effect of levodopa. 27) B - amyotrophic lateral sclerosis is a degenerative motor neuron disorder that affects muscles. Choice A describes Parkinson's disorder. Choice C describes multiple sclerosis Choice D describes guillain-barre's syndrome

28) A - regular pattern of activity/exercise and estrogen may increase HDL levels (good cholesterol). High density lipoprotein is necessary to prevent atherosclerosis. 29) B - GBS is an acute infectious polyneuritis of the cranial and peripheral nerves. It involves destruction of myelin sheath. It is usually preceded by a mild upper respiratory infection or gastroenteritis. It is characterized by ascending paralysis. Poor bladder tone is a characteristic manifestation. 30) B - the client might go to hypertensive crisis since he is not responding to medications. This increases risk to cerebrovascular accident (CVA). Therefore, this client should be given highest priority by the nurse. 31. The client experiences hypoglossal nerve damage. Which of the following assessment findings does the nurse expect in the client? a) difficulty of swallowing and protrusion of tongue b) asymmetry of the face c) severe pain on the side of the face d) inability to rotate the head and move shoulders 32. Which of the following indicates stimulation of the sympathetic nervous system (SNS)? a) hypotension b) urinary frequency c) diarrhea d) dilatation of pupils 33. A client with moderate Alzheimer's disease removes her clothes in the hall. Which of the following is the most appropriate nursing action? a) help the client put on her dress b) usher the client back to his room c) tell the client that such behavior is unacceptable d) remind her that when she undresses, she should do it inside her room 34. Which of the following nursing interventions should be included in the nursing care plan for the client with cerebral concussion? a) check leakage of cerebrospinal fluid through the nose b) check vital signs every 2 hours c) check neurologic status every 4 hours d) check pupillary reflexes once in each shift 35. The client who had cerebrovascular accident (CVA) has left-sided weakness. Which of the following instructions should be included regarding proper use of the cane? a) hold the cane on the right hand b) hold the cane on the left hand c) hold the cane alternately on each hand d) hold the cane with both hands

31) A - hypoglossal nerve provides motor nerve supply to the tongue. Hypoglossal nerve damage is characterized by difficulty of swallowing, protrusion of the tongue, deviation of the tongue to one side of the mouth. Asymmetry of the face is affectation of the facial nerve. Severe pain on the side of the face is affectation of the trigeminal nerve. Inability to rotate the head and move shoulders is affectation of the spinal accessory nerve. 32) D - SNS secretes norepinephrine and causes dilatation of pupils. Choices A, B, and C are effects of PNS. (SNS: everything is high and fast, except GI and GU). 33) D - a client with Alzheimer's disease experiences memory loss. Reminding the client will help him/her remember, e.g. undressing is done inside the room. 34) A - in head injury, it is very important to assess for CSF leakage. This indicates basilar head injury. This may lead to brainstem compression resulting to cardiopulmonary arrest. 35) A - use the cane on the stronger/unaffected area - the right hand. 36. Which of the following nursing diagnosis should be given highest priority by the nurse in a client diagnosed with Guillain-Barre Syndrome (GBS)? a) activity intolerance related to muscle weakness b) ineffective breathing pattern related to respiratory muscle weakness c) ineffective sexuality pattern related to paralysis d) ineffective coping related to body changes 37. A client has been diagnosed to have Alzheimer's disease. Which of the following is most appropriate nursing action to prevent sundowning syndrome? a) make the client stay in his room before dark b) turn lights on before dark c) feed the client before dark d) administer the client's medication before dark 38. Which of the following manifestations is most likely observed in a child withhydrocephalus? a) depressed anterior fontanel b) sunsetting eyes c) loud, vigorous cry d) short and thick neck 39. The client has been diagnosed to have ALS (amyotrophic lateral sclerosis). Which of the following manifestations characterize the disease? Select all that apply a) muscle weakness b) intention tremors c) muscle atrophy d) fatigue e) shuffling gait f) respiratory difficulty

ANSWERS AND RATIONALE

40. The client had undergone cerebral angiography. Which of the following potential complications should the nurse be most alert for? a) nausea and vomiting b) skin rashes c) hypertension d) hypotension

with the client in her room during an entire shift d) the staff members should allow the client to go out of his room only during meals times 44. Which of the following questions would evaluate that the client suffered fromfrontal lobe defect? a) what would you do if your house is on fire? b) what is the color of this flower? c) what sound do you hear now? d) what is the temperature of this fluid? 45. The charge nurse assigned the RN to a client with increased intracranial pressure. Which action by the RN needs intervention by the charge nurse? a) the RN places the client in semi-fowler's position b) the RN places the client in side-lying position with the neck flexed c) the RN places the client in supine position d) the RN places the client in side-lying, semi-fowler's position

NCLEX Secrets - Neurology Board Review: ANSWERS AND RATIONALE 36) B - in GBS, respiratory muscle weakness and paralysis occur. 37) B - turning lights on before dark prevents sundowning syndrome. 38) B - sunsetting eyes characterize hydrocephalus. Other signs and symptoms include: sudden enlargement of head, bulging fontanels, dilated scalp veins, separated sutures, Macewen sign, frontal enlargement/bossing, and thinning of skull bones. 39) A, C, D, F - amyotrophic lateral scerosis (ALS) is a motor neuron disorder. It is characterized by fatigue, awkwardness of fine finger movement, muscle wasting, dysphagia, muscle weakness, atrophy, fasciculations, dysarthria, jaw clonus, respiratory difficulty, spasticity of flexor muscles. This disease is also known as "Lou Gehrig's disease." 40) D - the contrast medium used in cerebral angiography has profound diuretic effect. Therefore, it may cause hypotension. 41. When lumbar puncture is done, the needle is inserted into which of the following intervertebral spaces? a) cervical 4 and 5 b) thoracic 4 and 5 c) lumbar 4 and 5 d) sacral 4 and 5 42. The cerebral cortex is responsible for regulating which of the following functions? a) motor activities, speech b) touch, temperature c) hearing, taste d) vision, reading 43. A client with Alzheimer's disease lives in a board and care facility. The client frequently wanders outside the care facility. Which of the following is the most essential measure to avoid injury in the client? a) the staff members should frequently orient the client b) the staff members should go for a walk with the client several times a day c) the staff members should assign a nursing assistant to stay

NCLEX Secrets - Neurology Board Review: ANSWERS AND RATIONALE 41) C - the spinal needle is inserted in L3 and L4, L4 and L5, L5 ans S1 intervertebral spaces to prevent trauma to the spinal cord, which ends at L1. 42) A - the cerebral cortex regulates motor activities, speech, logical operations (number skills, reasoning, scientific skills), insights, art awareness, imagination, music awareness, personality development, future planning. Choice B - parietal lobe is responsible for touch and temperature Choice C - the temporal lobe is responsible for hearing and taste Choice D - the occipital lobe is responsible for vision and hearing. 43) A - Alzheimer's disease is characterized by memory loss. Frequent orientation of the client is necessary. 44) A - frontal lobe is responsible for thought processes, logical operations, and reasoning. Choice B evaluates function of the occipital lobe. Choice C evaluates function of the temporal lobe. Choice D evaluates function of the parietal lobe. 45) B - the client with increased intracranial pressure should be positioned with the neck in neutral position. This prevents compression of jugular veins and will prevent cerebral venous congestion.

46. Which of the following assessment findings indicated increased intracranial pressure? Select all that apply

47) B a) headache b) tachycardia c) slow respiration d) narrowing of pulse pressure e) slow, bounding pulse f) hyperthermia 48) C 47. Which of the following nursing measures should be included when caring for a client with Parkinson's disease? a) put color on rails in going upstairs b) provide high toilet seat c) provide soft mattress d) apply restraints to reduce tremors 48. Michael suddenly went into seizures. To protect a child from injury the nurse should: a) retrain the child's arm and legs b) place a tongue blade in the child's mouth c) place a pillow under the child's head d) provide a waterproof pad for the bed 49. The first nursing priority when providing nursing care for Michael is to: a) administer antibiotic as ordered as soon as possible b) keep the room quiet and dim c) explain all the procedures to the patients d) begin low-flow oxygen per mask 50. Which room should the nurse assign to Ashley? a) room 201 with Joey, age 2 who underwent surgery for repair of hernia b) room 206 with Rica, age one who had pneumonia c) room 210 with Jack age 2 who has cerebral palsy d) room 214 with no roommate - the child should never be restrained during a seizure because such action by the nurse can cause fracture of the bones. Tongue blade must not be inserted when the child is already having a seizure because of the risk of injury. Placing waterproof pad at this time is no longer advisable. It is correct to slip pillow under the head to protect the head of the child from banging on any hard object. 49) A - the priority is to treat the patient with antibiotics at the soonest time possible because the longer the disease goes on without treatment, the greater the risk of seizures and of permanent neurologic damage, such as hearing loss, brain damage, blindness, loss of speech, learning disabilities and behavior problems. Non-neurologic complications may include kidney and adrenal gland failure. Bacterial infections of the central nervous system progress quickly. Within hours of the onset of symptoms, the disease can lead to shock and death. 50) D - Bacterial meningitis is a highly contagious. Therefore the patient must be placed in a private room to prevent transmitting it to other patients. Nurses who will be in contact with patients with meningitis should wear mask, the mask should be discarded right away when it gets moist or wet. The different types of bacterial meningitis are: - the client with Parkinson's disease experiences stiffness/rigidity due to inadequate dopamine production. Dopamine is a neurotransmitter that promotes muscle relaxation. High toilet seat facilitates the client's ability to sit during elimination.

Pneumococcus - this bacterium is the most common cause of meningitis in adults and children. It most often occurs when the bacterium Streptococcus pneumoniae (pneumococcus), the same bacterium that causes pneumonia and ear infections, enters the bloodstream and migrates to the brain and spinal cord. Mode of transmission is the same as pneumonia: droplet, direct contact and through respiratory discharges. Meningococcus - this bacterium is another common cause of meningitis in children under age 5, in teens and in young adults. Meningococcal meningitis commonly occurs when bloodstream. It's highly contagious and may cause localized epidemics in college dormitories, boarding schools and on military bases. Five strains of the Neisseria meningitidis bacterium cause meningococcal meningitis.Mode of transmission is by direct contact and respiratory droplets. This type of meningitis is communicable until the meningococci are no longer present from nose and mouth.Prophylactic treatment is Rifampicin taken for 4 days at a dose of 20 mg/kg/day. Haemophilus - the Haemophilus influenzae (H. influenzae) bacterium is the leading cause of

NCLEX Secrets - Neurology Board Review: ANSWERS AND RATIONALE 46) A, C, E, F - increased ICP is characterized by headache, nausea and vomiting, diplopia, increased systolic BP, slow respiration, slow bounding pulse, widening of pulse pressure, hyperthermia/hypothermia, altered LOC, papilledema, lateralizing sign.

bacterial meningitis in children under age 5. The use of the Hib vaccine prevents this type of meningitis. When it occurs, it tends to follow an upper respiratory infection, ear infection (otitis media) or sinusitis. Mode of transmission is by droplet infection and discharges from nose and throat during the infectious period. This type of meningitis is no longer communicable 48 hours after starting antibiotic therapy.

- falls is one of the greatest dangers among confused client. 52) D - a simple, structured environment should be provided to the client with Alzheimer's disease to help him cope up with his/her memory loss. 53) C - remove the client from the environment that may have triggered agitation. But continue provide care, like allowing him to finish his meal. 54) A - the natural end-product of protein is ammonia. Increased ammonia production causes hepatic encephalopathy. 55) A, B, C, D - the main problem with Alzheimer's disease is memory loss. Intention tremors and shuffling gait are manifestations of Parkinson's disease.

51. A 75-year old woman is admitted to a nursing home with a diagnosis of primary dementia of the Alzheimer's type. In the nursing home, which of these behaviors of the client is of greatest danger to her? a) she wanders into other patient's room b) she climbs over the side rails of her bed c) she eats the food off other resident's plates d) she refuses to change her clothes 52. An elderly client is diagnosed with Alzheimer's disease. When planning care, the nurse should include which of these vital considerations? a) allowing him to plan his own day b) encouraging outside diversional activities c) limiting his caloric intake d) providing a calm, predictable environment 53. While in the dining room having lunch, a nursing home client with Alzheimer's disease suddenly begins shouting and banging on the table. Which action should be taken by the nurse first? a) speak in a firm voice asking the client to stop the behavior b) put the client in the corner of the dining room by himself c) take the client back to his room to finish lunch d) remove the lunch tray until the client can control himself 54. A patient with hepatic encephalopathy is given instructions regarding his diet. Which of the following foods should the patient avoid? a) proteins b) calcium c) fats d) carbohydrates 55. A male client, 89 years old who has a mild Alzheimer's disease was admitted 2 days ago. Which of the following signs and symptoms are expected in the client? a) poor attention span b) poor personal hygiene c) unable to remember misplaced objects d) irritable e) intention tremors f) shuffling gait

56. Which of the following is a sign of neuroleptic malignant syndrome? a) Temperature of 39.9 C b) muscle weakness c) red-purplish blotchy areas in the skin d) weight gain 57. A nurse is caring for a client diagnosed to have head injury. Which of the following situations needs intervention by the nurse? a) the padded side rails are up b) the bed is adjusted to low level c) the client's spouse turns on the TV one hour in the afternoon and 2 hours in the evening d) the head of bed is elevated at 30 degree angle 58. Which of the following should be included when giving health teachings to a client with left hemianopsia? a) tell her to fully turn her head from side to side b) tell her to fully turn her head on her right side only c) tell her not to turn her head from side to side d) tell her to look forward most of the time 59. The client is diagnosed to have amyotrophic lateral sclerosis. Which of the following signs and symptoms should concern the nurse most? a) tremors b) difficulty in swallowing c) ptosis of the eyelid d) muscle atrophy 60. A client with closed head injury manifests increased urine output. What should the nurse do next? a) measure urine specific gravity b) start an IV fluid c) monitor blood glucose levels d) administer mannitol drip

NCLEX Secrets - Neurology Board Review: ANSWERS AND RATIONALE 51) B

NCLEX Secrets - Neurology Board Review: ANSWERS AND RATIONALE 56) A - fever is a characteristic manifestation of neuroleptic malignant syndrome 57) C - in a client with head injury, increased ICP occurs. External stimuli should be reduced to prevent seizures. 58) A - scanning technique will help the client with hemianopsia to see the entire visual field. 59) B - difficulty in swallowing may cause aspiration. This in turn may cause airway obstruction. 60) A - closed head injury causes increased intracranial pressure. This causes compression of the posterior pituitary gland that leads to hyposecretion of antidiuretic hormone (ADH). Diabetes insipidus occurs. Diabetes insipidus is characterized by polyuria and low urine specific gravity. 5) 4) ICP?

1. 2. 3. 4.

Urine output increases Pupils are 8 mm and nonreactive Systolic blood pressure remains at 150 mm Hg BUN and creatinine levels return to normal

Which of the following values is considered normal for

1. 2. 3. 4.

0 to 15 mm Hg 25 mm Hg 35 to 45 mm Hg 120/80 mm Hg

Which of the following symptoms may occur with a

phenytoin level of 32 mg/dl?

1. 2. 3.

Ataxia and confusion Sodium depletion Tonic-clonic seizure Urinary incontinence

Neuro ICP, LOC, meningitis 1) A client admitted to the hospital with a subarachnoid 6)

4.

Which of the following signs and symptoms of increased

hemorrhage has complaints of severe headache, nuchal rigidity, and projectile vomiting. The nurse knows lumbar puncture (LP) would be contraindicated in this client in which of the following circumstances?

ICP after head trauma would appear first?

1. 2. 3.

Bradycardia Large amounts of very dilute urine Restlessness and confusion Widened pulse pressure

1. 2. 3. 4. 2)

Vomiting continues Intracranial pressure (ICP) is increased The client needs mechanical ventilation Blood is anticipated in the cerebralspinal fluid (CSF) 7)

4.

Problems with memory and learning would relate to

which of the following lobes?

A client with a subdural hematoma becomes restless

1. 2. 3. 4.

Frontal Occipital Parietal Temporal

and confused, with dilation of the ipsilateral pupil. The physician orders mannitol for which of the following reasons?

1. 2. 3. 4.

To reduce intraocular pressure To prevent acute tubular necrosis To promote osmotic diuresis to decrease ICP To draw water into the vascular system to increase

8)

While cooking, your client couldnt feel the temperature

of a hot oven. Which lobe could be dysfunctional?

1. 2. 3. 4.

Frontal Occipital Parietal Temporal

blood pressure 3) A client with subdural hematoma was given mannitol to

decrease intracranial pressure (ICP). Which of the following results would best show the mannitol was effective?

9)

The nurse is assessing the motor function of an

13) The nurse is evaluating the status of a client who had a craniotomy 3 days ago. The nurse would suspect the client is developing meningitis as a complication of surgery if the client exhibits:

unconscious client. The nurse would plan to use which of the following to test the clients peripheral response to pain?

1. 2. 3. 4.

Sternal rub Pressure on the orbital rim Squeezing the sternocleidomastoid muscle Nail bed pressure 1. 2. 3. 4. A positive Brudzinskis sign A negative Kernigs sign Absence of nuchal rigidity A Glascow Coma Scale score of 15

10) The client is having a lumbar puncture performed. The nurse would plan to place the client in which position for the procedure?

14) A client is arousing from a coma and keeps saying, Just stop the pain. The nurse responds based on the knowledge that the human body typically and automatically responds to

1.

Side-lying, with legs pulled up and head bent down

pain first with attempts to:

onto the chest 2. 3. 4. Side-lying, with a pillow under the hip Prone, in a slight Trendelenburgs position Prone, with a pillow under the abdomen. 1. 2. 3. 4. Tolerate the pain Decrease the perception of pain Escape the source of pain Divert attention from the source of pain.

11) A nurse is assisting with caloric testing of the oculovestibular reflex of an unconscious client. Cold water is injected into the left auditory canal. The client exhibits eye conjugate movements toward the left followed by a rapid nystagmus toward the right. The nurse understands that this indicates the client has:

15) During the acute stage of meningitis, a 3-year-old child is restless and irritable. Which of the following would be most appropriate to institute?

1. 2.

Limiting conversation with the child Keeping extraneous noise to a minimum Allowing the child to play in the bathtub Performing treatments quickly

1. 2. 3. 4.

A cerebral lesion A temporal lesion An intact brainstem Brain death

3. 4.

16) Which of the following would lead the nurse to suspect that a child with meningitis has developed disseminated intravascular coagulation?

12) The nurse is caring for the client with increased intracranial pressure. The nurse would note which of the following trends in vital signs if the ICP is rising?

1. 2.

Hemorrhagic skin rash Edema Cyanosis Dyspnea on exertion

1.

Increasing temperature, increasing pulse, increasing

3. 4.

respirations, decreasing blood pressure. 2. Increasing temperature, decreasing pulse,

17) When interviewing the parents of a 2-year-old child, a history of which of the following illnesses would lead the nurse to suspect pneumococcal meningitis?

decreasing respirations, increasing blood pressure. 3. Decreasing temperature, decreasing pulse,

increasing respirations, decreasing blood pressure. 4. Decreasing temperature, increasing pulse, 1. 2. Bladder infection Middle ear infection

decreasing respirations, increasing blood pressure.

3. 4.

Fractured clavicle Septic arthritis

1.

Abnormal flexion of the upper extremities and

extension of the lower extremities 2. 3. 4. Rigid extension and pronation of the arms and legs Rigid pronation of all extremities Flaccid paralysis of all extremities

18) The nurse is assessing a child diagnosed with a brain tumor. Which of the following signs and symptoms would the nurse expect the child to demonstrate? Select all that apply.

22) Which of the following assessment data indicated nuchal 1. 2. 3. 4. 5. 6. Head tilt Vomiting Polydipsia Lethargy Increased appetite Increased pulse 1. 2. 3. 4. Positive Kernigs sign Negative Brudzinskis sign Positive homans sign Negative Kernigs sign rigidity?

19) A lumbar puncture is performed on a child suspected of having bacterial meningitis. CSF is obtained for analysis. A nurse reviews the results of the CSF analysis and determines that which of the following results would verify the diagnosis?

23) Meningitis occurs as an extension of a variety of bacterial infections due to which of the following conditions?

1. 2.

Congenital anatomic abnormality of the meninges Lack of acquired resistance to the various etiologic

1.

Cloudy CSF, decreased protein, and decreased

organisms 3. 4. Occlusion or narrowing of the CSF pathway Natural affinity of the CNS to certain pathogens

glucose 2. 3. 4. Cloudy CSF, elevated protein, and decreased glucose Clear CSF, elevated protein, and decreased glucose Clear CSF, decreased pressure, and elevated protein

24) Which of the following pathologic processes is often associated with aseptic meningitis?

20) A nurse is planning care for a child with acute bacterial meningitis. Based on the mode of transmission of this infection, which of the following would be included in the plan of care? 1. 2. 3. Ischemic infarction of cerebral tissue Childhood diseases of viral causation such as mumps Brain abscesses caused by a variety of pyogenic

organisms 1. No precautions are required as long as antibiotics 4. Cerebral ventricular irritation from a traumatic brain

have been started 2. 3. Maintain enteric precautions Maintain respiratory isolation precautions for at least

injury ANSWERS

24 hours after the initiation of antibiotics 4. Maintain neutropenic precautions

1.

2. Sudden removal of CSF results in pressures lower

in the lumbar area than the brain and favors herniation of the brain; therefore, LP is contraindicated with increased ICP. Vomiting may be caused by reasons other than increased ICP; therefore, LP isnt strictly contraindicated. An LP may be preformed on clients needing mechanical ventilation. Blood in the CSF is diagnostic for subarachnoid hemorrhage and was obtained before signs and symptoms of ICP.

21) A nurse is reviewing the record of a child with increased ICP and notes that the child has exhibited signs of decerebrate posturing. On assessment of the child, the nurse would expect to note which of the following if this type of posturing was present?

2.

3. Mannitol promotes osmotic diuresis by increasing

10. 1. The client undergoing lumbar puncture is positioned lying on the side, with the legs pulled up to the abdomen, and with the head bent down onto the chest. This position helps to open the spaces between the vertebrae.

the pressure gradient, drawing fluid from intracellular to intravascular spaces. Although mannitol is used for all the reasons described, the reduction of ICP in this client is a concern. 3. 1. Mannitol promotes osmotic diuresis by increasing

11. 3. Caloric testing provides information about differentiating between cerebellar and brainstem lesions. After determining patency of the ear canal, cold or warm water is injected in the auditory canal. A normal response that indicates intact function of cranial nerves III, IV, and VIII is conjugate eye movements toward the side being irrigated, followed by rapid nystagmus to the opposite side. Absent or dysconjugate eye movements indicate brainstem damage.

the pressure gradient in the renal tubes. Fixed and dilated pupils are symptoms of increased ICP or cranial nerve damage. No information is given about abnormal BUN and creatinine levels or that mannitol is being given for renal dysfunction or blood pressure maintenance. 4. 5. 1. Normal ICP is 0-15 mm Hg. 1. A therapeutic phenytoin level is 10 to 20 mg/dl. A

level of 32 mg/dl indicates toxicity. Symptoms of toxicity include confusion and ataxia. Phenytoin doesnt cause hyponatremia, seizure, or urinary incontinence. Incontinence may occur during or after a seizure. 6. 3. The earliest symptom of elevated ICP is a change 12. 2. A change in vital signs may be a late sign of increased intracranial pressure. Trends include increasing temperature and blood pressure and decreasing pulse and respirations. Respiratory irregularities also may arise.

in mental status. Bradycardia, widened pulse pressure, and bradypnea occur later. The client may void large amounts of very dilute urine if theres damage to the posterior pituitary. 7. 4. The temporal lobe functions to regulate memory 13. 1. Signs of meningeal irritation compatible with meningitis include nuchal rigidity, positive Brudzinskis sign, and positive Kernigs sign. Nuchal rigidity is characterized by a stiff neck and soreness, which is especially noticeable when the neck is fixed. Kernigs sign is positive when the client feels pain and spasm of the hamstring muscles when the knee and thigh are extended from a flexed-right angle position. Brudzinskis sign is positive when the client flexes the hips and knees in response to the nurse gently flexing the head and neck onto the chest. A Glascow Coma Scale of 15 is a perfect score and indicates the client is awake and alert with no neurological deficits.

and learning problems because of the integration of the hippocampus. The frontal lobe primarily functions to regulate thinking, planning, and judgment. The occipital lobe functions regulate vision. The parietal lobe primarily functions with sensory function. 8. 3. The parietal lobe regulates sensory function, which

would include the ability to sense hot or cold objects. The frontal lobe regulates thinking, planning, and judgment, and the occipital lobe is primarily responsible for vision function. The temporal lobe regulates memory. 9. 4. Motor testing on the unconscious client can be

14. 3. The clients innate responses to pain are directed initially toward escaping from the source of pain. Variations in individuals tolerance and perception of pain are apparent only in conscious clients, and only conscious clients are able to employ distraction to help relieve pain.

done only by testing response to painful stimuli. Nailbed pressure tests a basic peripheral response. Cerebral responses to pain are testing using sternal rub, placing upward pressure on the orbital rim, or squeezing the clavicle or sternocleidomastoid muscle.

15. 2. A child in the acute stage of meningitis is irritable and hypersensitive to loud noise and light. Therefore, extraneous

noise should be minimized and bright lights avoided as much as possible. There is no need to limit conversations with the child. However, the nurse should speak in a calm, gentle, reassuring voice. The child needs gentle and calm bathing. Because of the acuteness of the infection, sponge baths would be more appropriate than tub baths. Although treatments need to be completed as quickly as possible to prevent overstressing the child, any treatments should be performed carefully and at a pace that avoids sudden movements to prevent startling the child and subsequently increasing intracranial pressure.

21. 2. Decebrate posturing is characterized by the rigid extension and pronation of the arms and legs.

22. 1. A positive Kernigs sign indicated nuchal rigidity, caused by an irritative lesion of the subarachnoid space. Brudzinskis sign is also indicative of the condition.

23. 2. Extension of a variety of bacterial infections is a major causative factor of meningitis and occurs as a result of a lack of acquired resistance to the etiologic organisms. Preexisting CNS anomalies are factors that contribute to susceptibility.

16. 1. DIC is characterized by skin petechiae and a purpuric skin rash caused by spontaneous bleeding into the tissues. An abnormal coagulation phenomenon causes the condition.

24. 2. Aseptic meningitis is caused principally by viruses and is often associated with other diseases such as measles, mumps, herpes, and leukemia. Incidences of brain abscess are high in bacterial meningitis, and ischemic infarction of

17. 2. Organisms that cause bacterial meningitis, such as pneumococci or meningococci, are commonly spread in the body by vascular dissemination from a middle ear infection. The meningitis may also be a direct extension from the paranasal and mastoid sinuses. The causative organism is a pneumonococcus. A chronically draining ear is frequently also found.

cerebral tissue can occur with tubercular meningitis. Traumatic brain injury could lead to bacterial (not viral) meningitis.

1. The nurse explains that the neural synapse refers to the: 1. length of time it takes for afferent neurons to carry impulses to the CN 2. length of time it takes for efferent neurons to carry impulses to the m 3. space between the axons and dendrites of a neuron. 4. space between the axons of one neuron and the dendrites of the next

ANS: 4 Smooth coordinated transmission must travel from one neuron to anothe 18. 1, 2, 4. Head tilt, vomiting, and lethargy are classic signs assessed in a child with a brain tumor. Clinical manifestations are the result of location and size of the tumor. PTS: 1 DIF: Cognitive Level: Comprehension REF: 416 OBJ: N/A TOP: Anatomy and Physiology of Nervous System KEY: Nursing Process Step: Implementation MSC: NCLEX: Health Promotion and Maintenance

19. 2. A diagnosis of meningitis is made by testing CSF obtained by lumbar puncture. In the case of bacterial meningitis, findings usually include an elevated pressure, turbid or cloudy CSF, elevated leukocytes, elevated protein, and decreased glucose levels.

2. An older adult patient who is experiencing extreme stress rela demonstrate: 1. increased heart rate. 2. decreased blood pressure. 3. irregular respiration. 4. dilation of the pupils.

ANS: 4 Stress stimulates the fight-or-flight reaction, with release of epinephrine reduced peristalsis, and pupil dilation.

20. 3. A major priority of nursing care for a child suspected of having meningitis is to administer the prescribed antibiotic as soon as it is ordered. The child is also placed on respiratory isolation for at least 24 hours while culture results are obtained and the antibiotic is having an effect.

PTS: 1 DIF: Cognitive Level: Application REF: 418 OBJ: 1 TOP: Effects of Sympathetic Nervous System KEY: Nursing Process Step: Assessment MSC: NCLEX: Physiological Integ 3. The neurologic finding that would be considered abnormal in 1. smaller pupils, bilaterally. 2. bilateral slow pupillary response to light. 3. dizziness and problems with balance. 4. absence of Achilles tendon jerk. ANS: 3

Dizziness and vertigo, although common, are considered abnormal. PTS: 1 DIF: Cognitive Level: Analysis REF: 418 OBJ: 1 TOP: Age-Related Neurologic Changes KEY: Nursing Process Step: Assessment MSC: NCLEX: Health Promotion and Maintenance 4. The most reliable indicator of neurologic status is: 1. blood pressure. 2. pulse rate. 3. temperature. 4. level of consciousness.

MSC: NCLEX: Safe, Effective Care Environment

9. A patient with a severe head injury begins to assume a postur extremities. These assessments indicate: 1. increasing intracranial pressure with decorticate posturing. 2. decreasing intracranial pressure with decerebrate posturing. 3. decreasing intracranial pressure with decorticate posturing. 4. increasing intracranial pressure with decerebrate posturing.

ANS: 1 Increasing pressure on the tissue above the midbrain results in abnorma

PTS: 1 DIF: Cognitive Level: Analysis REF: 433-434 ANS: 4 OBJ: 4 TOP: Symptoms of Intracranial Pressure The ability to respond readily and correctly to person, place, and time is good evidence of intact sensorium. KEY: Nursing Process Step: Assessment MSC: NCLEX: Health Promotion and Maintenance PTS: 1 DIF: Cognitive Level: Knowledge REF: 421 OBJ: 2 TOP: Neurologic Assessment KEY: Nursing Process Step: Assessment MSC: NCLEX: Health Promotion and Maintenance

10. The nurse caring for a 90-year-old patient with a closed head 1. blood pressure change from 147/72 to 176/70. 2. respiration rate increase from 14 to 18. 3. slow pupillary reaction bilaterally. 5. The patient who is stuporous, but reacts by withdrawing from painful stimuli, is said to be: 4. temperature decrease from 100.2 to 97.6. 1. comatose. 2. lethargic. 3. semicomatose. ANS: 1 4. somnolent. The widening pulse pressure is an indicator of increased ICP. Respiration adults have a slowed pupillary response as they age. ANS: 3 PTS: 1 DIF: Cognitive Level: Application REF: 433 A stuporous patient who reacts to pain is semicomatose; no reaction to OBJ: 4 TOP: Nursing Care of Patient with Closed Head Injury pain is comatose. KEY: Nursing Process Step: Implementation PTS: 1 DIF: Cognitive Level: Analysis REF: 422 MSC: NCLEX: Physiological Integrity OBJ: 2 TOP: Neurologic Assessment KEY: Nursing Process Step: Assessment MSC: NCLEX: Physiological Integrity

11. A patient with generalized convulsive disorder has a nursing information about side effects of Dilantin. Which goal and outc 6. The nurse documents a normal finding from the Babinski reflex as a (the): 1. Absence of GI complaint; takes medication with food 1. downward curl of the toes. 2. Stimulate gingiva; brushes teeth vigorously to encourage gingival gro 2. big toe bending upward. 3. Maintain normal pattern of elimination; limits fluids and eats foods tha 3. spreading out of the toes. 4. Maintain normal sleep pattern; reduces stimuli and takes warm baths 4. pain in the big toe.

ANS: 1 ANS: 1 Dilantin is irritating toes tissues. Dilantin causes gingival hyperplasia, co Normal cortical function causes the toes to curl downward. Abnormal findings would be the to GI turning up and spreading. PTS: 1 DIF: Cognitive Level: Analysis PTS: 1 DIF: Cognitive Level: Application REF: 427 REF: 431, Drug Therapy table OBJ: 7 OBJ: 2 TOP: Neurologic Assessment TOP: Care Plan for Patient on Dilantin KEY: Nursing Process Step: Plannin KEY: Nursing Process Step: Assessment MSC: NCLEX: Health Promotion and Maintenance MSC: NCLEX: Health Promotion and Maintenance 12. for: 7. The nurse should assess the patient scheduled for an angiogram The patient with meningitis who has a positive Brudzinskis 1. flex the hips when the neck is flexed by the nurse. 1. dizziness. 2. not be able to extend the flexed leg fully due to hamstring pain. 2. allergy to shrimp. 3. resist efforts of the nurse to flex his or her neck. 3. increased blood pressure. 4. flex the big toe upward and fan out the other toes. 4. irregular heartbeat.

ANS: 1 ANS: 2 Allergy to shrimp and other shellfish also indicates a probable allergy toInflamed meninges will stimulate hip flexion to reduce meningeal discom the contrast medium. PTS: 1 DIF: Cognitive Level: Application REF: 442 PTS: 1 DIF: Cognitive Level: Analysis REF: 429 OBJ: 6 TOP: Symptoms of Meningitis OBJ: 2 TOP: Angiogram Preassessment KEY: Nursing Process Step: Assessment KEY: Nursing Process Step: Assessment MSC: NCLEX: Health Promotion and Maintenance MSC: NCLEX: Safe, Effective Care Environment 8. A patient who has a pacemaker might be ineligible for: 1. computed tomography (CT). 2. electromyography (EMG). 3. magnetic resonance imaging (MRI). 4. electroencephalography (EEG).

13. The assessment indicating that mannitol therapy for cerebra intracranial pressure (ICP) is: 1. increased blood pressure. 2. increased urinary output. 3. decreased pulse. 4. widening pulse pressure.

ANS: 3 Metal appliances may be affected by the magnetic field during an MRI. ANS: 2 Mannitol is a hyperosmolar diuretic that draws fluid from brain tissue int PTS: 1 DIF: Cognitive Level: Application REF: 430 Decreasing pulse and widening pulse pressure indicate increased ICP. OBJ: 2 TOP: Neurologic Assessment PTS: 1 DIF: Cognitive Level: Analysis REF: 434 KEY: Nursing Process Step: Assessment OBJ: 6 TOP: Mannitol Therapy in Increased ICP

KEY: Nursing Process Step: Assessment MSC: NCLEX: Health Promotion and Maintenance

KEY: Nursing Process Step: Planning MSC: NCLEX: Physiological Integrity

19. A Parkinsons patient is depressed because his drug protoco 14. To prevent a postlumbar puncture headache, the patient is instructed to lienurses response that would be the most helpful symptoms. The flat for at least: 1. 8 hours. 1. other drugs can be combined with L-dopa to increase its effectiveness 2. 6 hours. 2. the effect of these drugs has an uneven course; symptoms will begin 3. 4 hours. 3. the two drugs can be given in higher doses to control the symptoms. 4. 2 hours. 4. surgical interventions have been very effective in the control of parkin

ANS: 2 ANS: 1 Six hours is the optimum time in which the CSF fluid is regenerated to its prepuncture level. drugs to L-dopa may improve the conversion of L-d The addition of other had little effect on controlling the symptoms. PTS: 1 DIF: Cognitive Level: Comprehension REF: 423, Diagnostic Tests and 428 OBJ: 2 PTS: 1 DIF: Cognitive Level: Application REF: 447 TOP: Lumbar Puncture Care KEY: Nursing Process Step: ImplementationOBJ: 6 TOP: Treatment of Parkinsons Disease MSC: NCLEX: Physiological Integrity KEY: Nursing Process Step: Implementation MSC: NCLEX: Physiological Integrity 15. The nurse explains that the three symptoms characteristic 1. hypotension, tachycardia, and narrowing pulse pressure. 2. hypertension, tachycardia, and headache. 3. widening pulse pressure, headache, and seizure. 4. bradycardia, hypertension, and widening pulse pressure.

ANS: 4 Bradycardia, increasing blood pressure, and widening pulse pressure are all signs of increased ICP. ANS: 4 PTS: 1 DIF: Cognitive Level: Analysis REF: 434 Herbal remedies interfere with effectiveness of prescribed parkinsonian OBJ: 6 TOP: Increased ICP: Cushings Triad KEY: Nursing Process Step: Assessment MSC: NCLEX: Health Promotion and Maintenance

of Cushings triad associated with increased ICP are: 20. A patient with Parkinsons disease is considering taking St. J to Sinemet and L-dopa. The best response is to make the patien 1. depression is reduced by the use of herbal remedies such as St. John 2. doses of St. Johns wort and parkinsonian drugs should be taken on al 3. St. Johns wort must be taken in large doses to reduce depression. 4. herbal remedies can interfere with the effectiveness of the parkinsoni

PTS: 1 DIF: Cognitive Level: Application REF: 449 OBJ: 7 TOP: Treatment of Parkinsons Disease KEY: Nursing Process Step: Implementation MSC: NCLEX: Physiological Integrity 16. The nurse evaluates a positive outcome to the goal of teaching for the nursing diagnosis of: Knowledge deficit related to conservation of energy in a patient with multiple sclerosis when the patient says: 1. Now that I am taking steroids I will be able to work like I used to. 21. The nursing assessment that would indicate a need for sucti 2. Im making a list of things that are important and things I will simplyimpaired breathing patterns because of neuromuscular failure is have to let go. 3. I will make a plan to allow for long rest periods at least four times a 1. decreased pulse rate, respiration of 20. day. 4. I am working on balancing time between rest work and family time. increased pulse rate, adventitious breath sounds. 2. 3. increased pulse rate, respiration of 16. 4. decreased pulse, abdominal breathing. ANS: 4

Balancing time between various activities indicates that the MS patient understands the need to conserve energy, not just to give up things or attempt to perform at a preillness level. ANS: 2 Increased pulse rate, adventitious breath sounds, and abdominal breath PTS: 1 DIF: Cognitive Level: Application REF: 452

OBJ: 7 TOP: Altered Energy in MS Patients PTS: 1 DIF: Cognitive Level: Application REF: 446 KEY: Nursing Process Step: Evaluation MSC: NCLEX: Physiological Integrity 6 TOP: Nursing Care of Guillain-Barr Patient OBJ: KEY: Nursing Process Step: Assessment MSC: NCLEX: Physiological Integ

17. The patient in the ER states that she fell and hit her head and blacked out for a while, but became alert again. Suspecting an epidural hematoma, the nurse will be diligent in the A family member asks the nurse what would be an appropria 22. assessment of: 1. headache. useful reply would be: 2. increasing blood pressure. 1. soft-soled house shoes. 3. increasing respiration rate. 2. jigsaw puzzle. 4. vomiting. 3. set of card games. 4. satin sheets.

ANS: 2 Increasing blood pressure and a widening pulse pressure are indicators ANS: 4 of increased ICP. Satin sheets make moving in bed easier. Card games and jigsaw puzzles PTS: 1 DIF: Cognitive Level: Application REF: 439 support than soft-soled shoes. OBJ: 4 TOP: Epidural Hematoma Assessment KEY: Nursing Process Step: Implementation MSC: NCLEX: Physiological Integrity

PTS: 1 DIF: Cognitive Level: Application REF: 448 OBJ: 7 TOP: Care of Parkinsons KEY: Nursing Process Step: Planning MSC: NCLEX: Physiological Integrity

18. The intervention that should be added to the nursing care plan in supporting nutritional intake for a patient with Parkinsons disease is to: 23. When a patient falls to the floor in a generalized seizure, the 1. offer large meals with a variety of finger foods. 1. cradle the head to prevent injury. 2. thicken liquids to make them easier to swallow. 2. insert an object between the teeth to prevent the patient from biting 3. puree all foods and drink through a straw. 3. manually restrain the limbs. 4. offer a diet high in carbohydrates and fat and low in protein. 4. keep the patient on his or her back to prevent aspiration.

ANS: 2 ANS: 1 Thickened feedings are easier to swallow. Several small, protein-rich meals are preferableand turning it to the sidediet is unappealing. aspiratio Cradling the head to large ones. A pureed prevents injury and mouth often result in injury. PTS: 1 DIF: Cognitive Level: Application REF: 449 OBJ: 7 TOP: Nutrition in Parkinsons Disease PTS: 1 DIF: Cognitive Level: Application REF: 437, Box 27-3 OBJ: 7 TOP: Seizure Implementations

KEY: Nursing Process Step: Implementation MSC: NCLEX: Physiological Integrity

changes.

PTS: 1 DIF: Cognitive Level: Analysis REF: 418 OBJ: 7 TOP: Age-Related Cerebral Changes 24. In caring for the postcraniotomy patient, the nurse can helpKEY: Nursing Process Step: Planning reduce ICP by: MSC: NCLEX: Health Promotion and Maintenance 1. keeping the patient flat in bed. 2. elevating the head of the bed 30 degrees. 3. closely monitoring the IV rate. OTHER 4. turning the patient to her or his right side. 1. The nurse assessing the level of consciousness in a patient w the most complex): ANS: 2 1. Apply pressure to the nailbed. Elevating the head of the bed at least 30 degrees helps with reducing ICP.Shake patient. 2. PTS: 1 DIF: Cognitive Level: Application REF: 430 3. Touch the patient. OBJ: 6 TOP: Intervening for increased ICP 4. Call the patients name. KEY: Nursing Process Step: Implementation 5. Approach the patient. MSC: NCLEX: Physiological Integrity

ANS: 5, 4, 3, 2, 1 25. The nurse updates the nursing care plan for a patient with amyotrophic lateral sclerosis who is uniquely prone to The assessment begins with simply approaching the patient and progres depression because: 1. nutritional intake is poor. PTS: 1 DIF: Cognitive Level: Application REF: 422 2. intellectual capacity is not affected. OBJ: 2 TOP: Assessing Level of Consciousness 3. mobility is limited. KEY: Nursing Process Step: Implementation 4. communication is altered. MSC: NCLEX: Physiological Integrity

ANS: 2 2. The nurse conducting a Romberg test will ask the patient to ( Because of their unimpaired intellect, ALS patients are able to assess theirtouch his or her nose increases their finger with the eyes open. 1. deterioration, which with the index risk for depression. Altered mobility, nutrition, and communication are common to many disorders. with eyes closed. 2. stand PTS: 1 DIF: Cognitive Level: Application REF: 462 3. touch his or her nose with the index finger with the eyes closed. OBJ: 6 TOP: Symptoms of ALS 4. touch his or her fingertip to nurses fingertip. KEY: Nursing Process Step: Assessment MSC: NCLEX: Psychosocial Integrity the knees with the palms and then the back of the hands rapidly. 5. pat ANS: 2, 5, 4, 1, 3 1. The nurse explains that neurotransmitters, which support smooth simple exercises help assess(select all that apply): These neural transmission, are balance and perception. 1. acetylcholine. PTS: 1 DIF: Cognitive Level: Analysis REF: 426 2. cerebrospinal fluid. OBJ: 2 TOP: Romberg Test for Balance 3. dopamine. KEY: Nursing Process Step: Assessment 4. dendrite. MSC: NCLEX: Health Promotion and Maintenance 5. epinephrine. MULTIPLE RESPONSE ANS: 1, 3, 5 Acetylcholine, dopamine, and epinephrine are neurotransmitters. CSF bathes post-CVA patient who has has no transmissionright 1. A the brain and spinal cord but weakness on the activity; the dendrite is the locus of the synapse. side and impaired reasoning has had the CVA in the: PTS: 1 DIF: Cognitive Level: Comprehension REF: 416 1. left hemisphere of the cerebrum. OBJ: 1 TOP: Neurotransmitters 2. right hemisphere of the cerebrum. KEY: Nursing Process Step: Implementation 3. left cerebellum. MSC: NCLEX: Health Promotion and Maintenance 4. right cerebellum. 2. The nurse assesses a K+ level of 6.2 mEq/L and becomes alert for the associated sign of (select all that apply): ANS: 1 1. pain on movement. Impaired motor strength on the right side in conjunction with 2. contractions of the hands. 3. twitching of muscles. impaired reasoning indicates a lesion in the left hemisphere of 4. lethargy. the cerebrum. The cerebellum controls balance and is not 5. trembling of limbs. contralateral. PTS: 1 DIF: Cognitive Level: Analysis REF: 460 ANS: 3, 5 Increased levels of K+ cause twitching and tremor related to the alteredOBJ: 3 TOP: Symptoms of CVAfor smooth organized function. depolarization that provides KEY: Nursing Process Step: Assessment MSC: NCLEX: PTS: 1 DIF: Cognitive Level: Analysis REF: 415 Physiological Integrity OBJ: 6 TOP: K+ Excess KEY: Nursing Process Step: Assessment MSC: NCLEX: Health Promotion and Maintenance 2. The nurse counsels that the patient at the greatest 3. The nurse organizes the plan of care with regard to normal brain alterations associated with age, which are (select all risk for a CVA is a: that apply): 1. 20-year-old obese Latin woman on birth control pills. 1. decrease in brain weight. 2. pigmentation of brain with lipofuscin. 2. 40-year-old athletic white man with a family history of CVA. 3. present of amyloid. 3. 60-year-old Asian woman who smokes occasionally. 4. tiny clot formation. 4. 65-year-old African-American man with hypertension. 5. tangled nerve fibers. ANS: 4 ANS: 1, 2, 3, 5 All listed are expected changes that affect the older adults neurologic function except for tiny clot formations, which are of CVA than Older African-Americans have a higher incidence pathologic

occasional smokers, young, or athletic persons. Hypertension increases the risk. PTS: 1 DIF: Cognitive Level: Analysis REF: 462 OBJ: 1 TOP: CVA Risk Factors KEY: Nursing Process Step: Assessment MSC: NCLEX: Health Promotion and Maintenance

PTS: 1 DIF: Cognitive Level: Application REF: 464 OBJ: 1 TOP: Post-TIA Teaching KEY: Nursing Process Step: Planning MSC: NCLEX: Health Promotion and Maintenance

3. A patient experienced a period of momentary confusion, dizziness, and slurred speech, but recovered in 2 hours. The most helpful assessment in the diagnosis of this episode would be: 1. patient complaint of nausea. 2. blood pressure of 140/90. 3. patient complaint of headache. 4. auscultation of a bruit over the carotid. ANS: 4 A carotid bruit is evidence of a narrowing in that vessel, a symptom of a possible CVA or TIA. BP of 140/90 is within normal limits. Headache and nausea alone are too common to be definitive. PTS: 1 DIF: Cognitive Level: Application REF: 464 OBJ: 2 TOP: TIA Diagnosis KEY: Nursing Process Step: Assessment MSC: NCLEX: Health Promotion and Maintenance

6. The patient recovering from a CVA asks the purpose of the Coumadin (warfarin). The best response by the nurse is that Coumadin: 1. dissolves the clot. 2. prevents formation of new clots. 3. dilates the vessels to improve blood flow. 4. suppresses the formation of platelets. ANS: 2 Coumadin and heparin prevent more clots rather than dissolving them. Coumadin has no effect on vasodilation or blood cell production. PTS: 1 DIF: Cognitive Level: Comprehension REF: 464 OBJ: 3 TOP: Coumadin Therapy KEY: Nursing Process Step: Implementation MSC: NCLEX: Health Promotion and Maintenance

4. The patient who experienced a TIA is placed on warfarin (Coumadin) and has laboratory reports reflecting a therapeutic range for that drug, which are: 1. prothrombin time (PT), 35 seconds; control (normal), 20 seconds; INR, 2. 2. partial thromboplastin time (PTT), 30 seconds; control (normal), 30 seconds. 3. prothrombin time (PT), 45 seconds; control (normal), 20 seconds; INR, 4. 4. partial thromboplastin time (PTT), 52 seconds; control (normal), 30 seconds. ANS: 1 Prothrombin time of 1.5 to 2 times normal is the therapeutic goal for Coumadin. The INR for Coumadin therapy is 2 to 3. Partial thromboplastin time is the test for heparin. PTS: 1 DIF: Cognitive Level: Analysis REF: 465, Drug Therapy table OBJ: 3 TOP: Anticoagulant Therapy KEY: Nursing Process Step: Assessment MSC: NCLEX: Physiological Integrity

7. The nurse explains that a patient who has been determined to have had a complete stroke as a result of a ruptured vessel in the left hemisphere would be classified as: 1. ischemic, embolic. 2. hemorrhagic, subarachnoid. 3. hemorrhagic, intracerebral. 4. ischemic, thrombotic. ANS: 3 A ruptured vessel in a hemisphere is an intracerebral hemorrhagic CVA. It is not in the subarachnoid space. Ischemic CVAs are due to occluded vessels. PTS: 1 DIF: Cognitive Level: Analysis REF: 466 OBJ: 2 TOP: CVA Classification KEY: Nursing Process Step: Assessment MSC: NCLEX: Physiological Integrity

8. Immediately after a CVA, a major nursing priority is ensuring: 1. preservation of motor function. 2. airway maintenance. 3. adequate hydration. 4. control of elimination. ANS: 2 Adequate oxygenation prevents hypoxemia, which can extend and worsen the CVA. PTS: 1 DIF: Cognitive Level: Application REF: 470 OBJ: 6 TOP: Nursing Care of Acute CVA KEY: Nursing Process Step: Planning MSC: NCLEX: Physiological Integrity

5. The nurse updates the teaching plan for a post-TIA patient to include the provision for: 1. a daily aspirin dose. 2. long rest periods daily. 3. reduction of fluid intake to 800 mL/day. 4. high-carbohydrate diet. ANS: 1 Daily aspirin reduces platelet aggregation and may prevent another attack. Reduction of fluid and long rest periods encourage clot formation.

9. The nurse recognizes that the acute phase of a CVA has ended when: 1. 48 hours has passed from onset. 2. the patient begins to respond verbally.

3. the blood pressure drops. 4. vital signs and neurologic signs stabilize. ANS: 4 When the vital and neurologic signs stabilize, the acute phase has ended. Verbal response, lower BP, and passage of time, without other signs, are not adequate evidence. PTS: 1 DIF: Cognitive Level: Analysis REF: 469 OBJ: 6 TOP: Acute Phase of CVA KEY: Nursing Process Step: Assessment MSC: NCLEX: Physiological Integrity

Blood in the spinal fluid indicates hemorrhagic stroke and will help direct medical protocol in subsequent treatment. PTS: 1 DIF: Cognitive Level: Comprehension REF: 469 OBJ: 5 TOP: CVA Diagnostic Tests KEY: Nursing Process Step: N/A MSC: NCLEX: Health Promotion and Maintenance

10. A patient in the acute phase of a stroke who has been speaking distinctly begins to speak indistinctly and only with great effort, but is coherent. The nurse assesses: 1. stroke in evolution with dysarthria. 2. lacunar stroke with fluent aphasia. 3. complete stroke with global aphasia. 4. stroke in evolution with dyspraxia. ANS: 1 As symptoms worsen, the CVA is still evolving. Speech that is coherent but difficult is dysarthria rather than any type of aphasia. Dyspraxia is a motor impairment, not speech impairment. PTS: 1 DIF: Cognitive Level: Analysis REF: 466 OBJ: 4 TOP: CVA Deficits KEY: Nursing Process Step: Assessment MSC: NCLEX: Health Promotion and Maintenance

13. A patient who has suffered a hemorrhagic stroke is placed on a protocol of 60 mg nimodipine (calcium channel blocker) every 4 hours. The patients pulse is 82 prior to the administration of the prescribed dose. The nurse should: 1. give the full dose as prescribed, without further assessment. 2. omit the dose, recording the pulse rate as the rationale. 3. delay the dose until the pulse is below 60. 4. give half of the prescribed dose (30 mg). ANS: 1 The dose should be given; it would be held only if the pulse is below 60. Assessments should be made regarding BP, urine output, and edema. PTS: 1 DIF: Cognitive Level: Application REF: 465, Drug Therapy table OBJ: 3 TOP: CVA Medical Protocol KEY: Nursing Process Step: Implementation MSC: NCLEX: Physiological Integrity

11. Several days after a CVA, the patients family asks the nurse if tissue plasminogen activator (tPA) is a drug therapy option now. The nurses response is based on the knowledge that this drug must be used within: 1. 3 hours of onset of symptoms. 2. 5 hours of onset of symptoms. 3. 10 hours of onset of symptoms. 4. 24 hours of onset of symptoms. ANS: 1 tPA is to be given within 3 hours of onset of symptoms per FDA guidelines. PTS: 1 DIF: Cognitive Level: Comprehension REF: 470 OBJ: 3 TOP: CVA Medication Implementation KEY: Nursing Process Step: Implementation MSC: NCLEX: Health Promotion and Maintenance

14. During the acute CVA phase, there is a risk for falls related to paralysis. The intervention that best protects the patient from injury is: 1. keep the bed in a high position for ease of nursing care. 2. keep the side rails up, according to agency policy. 3. assess vision deficit related to ptosis. 4. monitor the condition every 2 hours. ANS: 2 Rails keep patients in bed. The bed should be low, monitoring the patient should be more frequent than every 2 hours, and visual assessment is not directly related to fall prevention. PTS: 1 DIF: Cognitive Level: Application REF: 476 OBJ: 7 TOP: Acute Care: Fall Prevention KEY: Nursing Process Step: Implementation MSC: NCLEX: Safe, Effective Care Environment

12. The nurse explains that a lumbar puncture is most helpful as a diagnostic tool for a new stroke patient, because it can help determine if the stroke: 1. is lacunar. 2. is hemorrhagic or embolic. 3. is complete or in evolution. 4. will result in paralysis. ANS: 2

15. Pneumonia is the most frequent cause of death after a stroke. The intervention that would be contraindicated in the acute care of a patient with a hemorrhagic CVA is to: 1. thicken liquids to ease swallowing and prevent aspiration. 2. change position every 30 to 60 minutes. 3. maintain adequate fluid intake, orally or IV. 4. encourage forceful coughing to stimulate deep breathing. ANS: 4 Forceful coughing is contraindicated for a hemorrhagic CVA patient, because it may cause ICP. PTS: 1 DIF: Cognitive Level: Application REF: 476 OBJ: 7 TOP: Prevention of Pneumonia

KEY: Nursing Process Step: Implementation MSC: NCLEX: Physiological Integrity

16. The assessments that indicate a fluid volume excess in a patient in the acute phase of a CVA is: 1. decreased blood pressure. 2. weak pulse. 3. adventitious breath sounds. 4. high specific gravity of urine. ANS: 3 Crackles in the lung fields are a major indicator of fluid excess. The pulse and BP are elevated in fluid excess. Urine specific gravity is low in fluid excess. PTS: 1 DIF: Cognitive Level: Analysis REF: 477 OBJ: 7 TOP: Fluid Excess KEY: Nursing Process Step: Assessment MSC: NCLEX: Physiological Integrity

1. the blood pressure has been within normal limits for 24 hours. 2. the patient makes positive statements about his condition. 3. there are no further neurologic deficits observed. 4. there are successful attempts at independent function. ANS: 3 When no further deficits are noted and all vital signs have stabilized, the patient is considered to be in the rehabilitation phase. Positive statements and attempts at independence are not sufficient. PTS: 1 DIF: Cognitive Level: Analysis REF: 480 OBJ: 7 TOP: Rehabilitation Phase KEY: Nursing Process Step: Assessment MSC: NCLEX: Physiological Integrity

17. The nurse adds to the nursing care plan the intervention that will help preserve joint mobility in the acute phase of a CVA, which is: 1. pull the limbs on the affected side into a functional position. 2. aggressive full range of motion for all extremities. 3. support affected points in good functional alignment. 4. exercise the limbs every 8 hours. ANS: 3 Limbs maintained in a functional anatomical position and gently exercised (never pulled) into an acceptable range of motion several times a shift will maintain optimal mobility. PTS: 1 DIF: Cognitive Level: Application REF: 479 OBJ: 7 TOP: Preserving Joint Mobility KEY: Nursing Process Step: Implementation MSC: NCLEX: Physiological Integrity

20. In the rehabilitation phase of a CVA, patients with homonymous hemianopsia need to have their environment arranged so that persons approaching and important items are available on: 1. the unaffected side. 2. the affected side. 3. the direct front. 4. either side. ANS: 2 Making the patient scan the affected side helps stimulate the return of normal function in the rehabilitation phase. PTS: 1 DIF: Cognitive Level: Application REF: 469 OBJ: 7 TOP: Hemianopsia KEY: Nursing Process Step: Planning MSC: NCLEX: Physiological Integrity

18. The postembolic CVA patient in the acute phase has an order for 400 units of heparin per hour IV. The heparin is in a solution of 5000 units/100 mL NS. The nurse should set the electronic IV monitor at how many milliliters per hour? 1. 6 2. 8 3. 10 4. 16 ANS: 2 Regardless of the method of calculation, there are 50 units of heparin in each milliliter of the solution; 8 mL/hour delivers 400 units. (5000 units divided by 100 mL NS = 50 units/mL. 400 units divided by 50 units/mL = 8 mL.) PTS: 1 DIF: Cognitive Level: Application REF: N/A review for application of prior knowledge OBJ: 7 TOP: Heparin Therapy KEY: Nursing Process Step: Implementation MSC: NCLEX: Physiological Integrity

21. The nurse, using the nursing diagnosis Imbalanced nutrition related to dysphagia, with the goal of adequate nutrition, would select the appropriate outcome criterion as: 1. offers variety of food groups. 2. eats half of all meals offered. 3. maintains body weight of 150 to 155 pounds. 4. eats all meals independently. ANS: 3 The maintenance of a desired weight is indicative of adequate nutrition. Eating a portion of meal or eating independently does not adequately measure the extent the goal was met. Offering a variety of foods is a nursing or dietary function, not an outcome. PTS: 1 DIF: Cognitive Level: Analysis REF: 477 OBJ: 8 TOP: Rehabilitation: Nutrition KEY: Nursing Process Step: Planning MSC: NCLEX: Health Promotion and Maintenance

19. The nurse assesses that the CVA patient is in transition to the rehabilitation phase when:

22. The nurse selects the most effective intervention for best support of regular bowel elimination and prevention of constipation, which is: 1. limit fluid intake to 32 to 50 ounces/day to compact stool. 2. small soapsuds enema every other day to cleanse bowel. 3. daily stool softeners, with consistent time to attempt

elimination. 4. a strong laxative on a daily basis to encourage evacuation. ANS: 3 Daily stool softeners rather than daily laxatives or frequent enemas help restore regularity and bowel tone. PTS: 1 DIF: Cognitive Level: Analysis REF: 479 OBJ: 7 TOP: Bowel Elimination KEY: Nursing Process Step: Planning MSC: NCLEX: Health Promotion and Maintenance

4. discharge to home with scheduled visits from an occupational therapist. ANS: 1 A rehabilitation center with all modalities of support (physical therapy, occupational therapy, speech therapy, and simulated home environments) is obviously the best option. PTS: 1 DIF: Cognitive Level: Comprehension REF: 484 OBJ: 9 TOP: Postdischarge Planning KEY: Nursing Process Step: Planning MSC: NCLEX: Safe, Effective Care Environment

23. The post-CVA patient in the rehabilitation phase accidentally knocks the adapted plate from the table and bursts into tears after failing to feed himself. The nurses best response would be: 1. Dont cry. Youll be mastering eating in no time. 2. I dont believe crying will help. Lets try drinking from a special cup. 3. Bless your heart! Let me get a new meal and feed you. 4. Learning new skills is hard. Lets see what may have caused the trouble. ANS: 4 Recognizing effort and showing support are the best approaches to depression and frustration. Babying the patient and admonitions against crying add to the problem. PTS: 1 DIF: Cognitive Level: Analysis REF: 481-482 OBJ: 7 TOP: Rehabilitation: Coping KEY: Nursing Process Step: Implementation MSC: NCLEX: Psychosocial Integrity

26. The nurse explains that a lacunar stroke differs from an ischemic CVA in that a lacunar CVA (select all that apply): 1. causes a great deal of pain. 2. alters the personality. 3. affects small arteries. 4. nearly always results in blindness. 5. produces larger amount of neurologic damage. ANS: 3 The lacunar CVA only affects small arteries and produces a small amount of neurologic damage. PTS: 1 DIF: Cognitive Level: Analysis REF: 467 OBJ: 4 TOP: Lacunar CVA KEY: Nursing Process Step: Implementation MSC: NCLEX: Physiological Integrity

MULTIPLE RESPONSE 1. The nurse suspects a TIA when the assessment of a patient reveals transitory (select all that apply): 1. incontinence. 2. dysphagia. 3. ptosis. 4. tinnitus. 5. dysarthria. ANS: 2, 3, 4, 5 All but transitory incontinence are classic symptoms of TIA. These deficits usually disappear without permanent disability in about 24 hours. PTS: 1 DIF: Cognitive Level: Comprehension REF: 462-463 OBJ: 1 TOP: Symptoms of TIA KEY: Nursing Process Step: Assessment MSC: NCLEX: Physiological Integrity

24. The instruction that is most helpful in teaching the rehabilitating CVA patient and his family about altered sensation is to: 1. make frequent assessments for signs of pressure or injury. 2. use the affected side in supporting the patient in ambulation and transfer to stimulate better sensation. 3. apply ice packs to the affected limbs to encourage return of sensation. 4. apply a heating pad to the affected limbs to increase circulation. ANS: 1 Frequent assessment will allow early detection. Use of hot or cold applications and using the affected limbs in transfer or ambulation may cause injury. PTS: 1 DIF: Cognitive Level: Application REF: 484 OBJ: 7 TOP: Altered Sensation KEY: Nursing Process Step: Planning MSC: NCLEX: Safe, Effective Care Environment

25. The nurse encourages the posthospital option that would provide the most comprehensive assistance to a recovering post-CVA, which is: 1. transfer to a rehabilitation center. 2. discharge to home with scheduled visits from home health nurses. 3. discharge to home with scheduled visits from a physical therapist.

2. The nurse explains that the purpose of a stent in the carotid artery of a person with a TIA is to (select all that apply): 1. capture circulating clots. 2. help with subsequent angioplasties. 3. keep the artery open. 4. prevent hemorrhage. 5. measure pressure in the artery. ANS: 3 The only thing that the stent does is keep the artery open.

PTS: 1 DIF: Cognitive Level: Analysis REF: 466 OBJ: 6 TOP: Use of Stent KEY: Nursing Process Step: Implementation MSC: NCLEX: Physiological Integrity

3. The nurse is supportive of the frustrated patient with expressive aphasia because the condition is characterized by (select all that apply): 1. speech that sounds normal, but makes no sense. 2. total inability to communicate. 3. difficulty understanding the written and spoken word. 4. stuttering and spitting. 5. difficulty initiating speech. ANS: 5 Expressive aphasia makes it difficult for the patient to initiate speech. PTS: 1 DIF: Cognitive Level: Analysis REF: 468 OBJ: 3 TOP: Expressive Aphasia KEY: Nursing Process Step: Implementation MSC: NCLEX: Psychosocial Integrity

COMPLETION 1. The nurse checks the O2 in the circulating volume for adequate concentration to support the brains need of ____________________% of the oxygen supply of the body. ANS: 20% PTS: 1 DIF: Cognitive Level: Application REF: 461 OBJ: 6 TOP: O2 Needs of the Brain KEY: Nursing Process Step: Implementation MSC: NCLEX: Physiological Integrity

Das könnte Ihnen auch gefallen